Create all numbers from 1-100 using 1,3,3,6












12












$begingroup$


Create all the numbers from $1$ to $100$ using the numbers $1$,$3$,$3$, and $6$.




  • You can only use each number once, except for the $3$, of which you have two.

  • You can use addition ($x+y$), subtraction ($x-y$), division ($frac{x}{y}$), multiplication ($xtimes y$), exponentiation ($x^y$) and roots ($sqrt[leftroot{-2}uproot{2}x]{y}$).

  • You can combine numbers like $1$ and $3$ to $13$ etc.

  • You must use all numbers.
    EDIT: no factorials, in squareroots 2 is hidden, no combining results of operations, you can use parentheses and start with negative numbers, no rounding and no decimal points. Good Luck










share|improve this question











$endgroup$








  • 4




    $begingroup$
    Can we combine the results of operations? For example, is $(1+3) | 36 = 436$ (where | indicates concatenation)
    $endgroup$
    – Hugh
    Jan 27 at 22:14






  • 1




    $begingroup$
    If we need to take a square root, is the two implied?
    $endgroup$
    – Hugh
    Jan 27 at 22:14






  • 1




    $begingroup$
    can we use factorial?
    $endgroup$
    – Omega Krypton
    Jan 27 at 23:23






  • 3




    $begingroup$
    Factorials are most likely out, but what about parentheses, unary minus (like starting with -1) and decimal points?
    $endgroup$
    – Bass
    Jan 28 at 0:25








  • 1




    $begingroup$
    If decimal is allowed then round would probably valid too?
    $endgroup$
    – Mukyuu
    Jan 28 at 3:31
















12












$begingroup$


Create all the numbers from $1$ to $100$ using the numbers $1$,$3$,$3$, and $6$.




  • You can only use each number once, except for the $3$, of which you have two.

  • You can use addition ($x+y$), subtraction ($x-y$), division ($frac{x}{y}$), multiplication ($xtimes y$), exponentiation ($x^y$) and roots ($sqrt[leftroot{-2}uproot{2}x]{y}$).

  • You can combine numbers like $1$ and $3$ to $13$ etc.

  • You must use all numbers.
    EDIT: no factorials, in squareroots 2 is hidden, no combining results of operations, you can use parentheses and start with negative numbers, no rounding and no decimal points. Good Luck










share|improve this question











$endgroup$








  • 4




    $begingroup$
    Can we combine the results of operations? For example, is $(1+3) | 36 = 436$ (where | indicates concatenation)
    $endgroup$
    – Hugh
    Jan 27 at 22:14






  • 1




    $begingroup$
    If we need to take a square root, is the two implied?
    $endgroup$
    – Hugh
    Jan 27 at 22:14






  • 1




    $begingroup$
    can we use factorial?
    $endgroup$
    – Omega Krypton
    Jan 27 at 23:23






  • 3




    $begingroup$
    Factorials are most likely out, but what about parentheses, unary minus (like starting with -1) and decimal points?
    $endgroup$
    – Bass
    Jan 28 at 0:25








  • 1




    $begingroup$
    If decimal is allowed then round would probably valid too?
    $endgroup$
    – Mukyuu
    Jan 28 at 3:31














12












12








12


5



$begingroup$


Create all the numbers from $1$ to $100$ using the numbers $1$,$3$,$3$, and $6$.




  • You can only use each number once, except for the $3$, of which you have two.

  • You can use addition ($x+y$), subtraction ($x-y$), division ($frac{x}{y}$), multiplication ($xtimes y$), exponentiation ($x^y$) and roots ($sqrt[leftroot{-2}uproot{2}x]{y}$).

  • You can combine numbers like $1$ and $3$ to $13$ etc.

  • You must use all numbers.
    EDIT: no factorials, in squareroots 2 is hidden, no combining results of operations, you can use parentheses and start with negative numbers, no rounding and no decimal points. Good Luck










share|improve this question











$endgroup$




Create all the numbers from $1$ to $100$ using the numbers $1$,$3$,$3$, and $6$.




  • You can only use each number once, except for the $3$, of which you have two.

  • You can use addition ($x+y$), subtraction ($x-y$), division ($frac{x}{y}$), multiplication ($xtimes y$), exponentiation ($x^y$) and roots ($sqrt[leftroot{-2}uproot{2}x]{y}$).

  • You can combine numbers like $1$ and $3$ to $13$ etc.

  • You must use all numbers.
    EDIT: no factorials, in squareroots 2 is hidden, no combining results of operations, you can use parentheses and start with negative numbers, no rounding and no decimal points. Good Luck







calculation-puzzle formation-of-numbers






share|improve this question















share|improve this question













share|improve this question




share|improve this question








edited Jan 28 at 10:19







Michał Uraszewski

















asked Jan 27 at 22:00









Michał UraszewskiMichał Uraszewski

6116




6116








  • 4




    $begingroup$
    Can we combine the results of operations? For example, is $(1+3) | 36 = 436$ (where | indicates concatenation)
    $endgroup$
    – Hugh
    Jan 27 at 22:14






  • 1




    $begingroup$
    If we need to take a square root, is the two implied?
    $endgroup$
    – Hugh
    Jan 27 at 22:14






  • 1




    $begingroup$
    can we use factorial?
    $endgroup$
    – Omega Krypton
    Jan 27 at 23:23






  • 3




    $begingroup$
    Factorials are most likely out, but what about parentheses, unary minus (like starting with -1) and decimal points?
    $endgroup$
    – Bass
    Jan 28 at 0:25








  • 1




    $begingroup$
    If decimal is allowed then round would probably valid too?
    $endgroup$
    – Mukyuu
    Jan 28 at 3:31














  • 4




    $begingroup$
    Can we combine the results of operations? For example, is $(1+3) | 36 = 436$ (where | indicates concatenation)
    $endgroup$
    – Hugh
    Jan 27 at 22:14






  • 1




    $begingroup$
    If we need to take a square root, is the two implied?
    $endgroup$
    – Hugh
    Jan 27 at 22:14






  • 1




    $begingroup$
    can we use factorial?
    $endgroup$
    – Omega Krypton
    Jan 27 at 23:23






  • 3




    $begingroup$
    Factorials are most likely out, but what about parentheses, unary minus (like starting with -1) and decimal points?
    $endgroup$
    – Bass
    Jan 28 at 0:25








  • 1




    $begingroup$
    If decimal is allowed then round would probably valid too?
    $endgroup$
    – Mukyuu
    Jan 28 at 3:31








4




4




$begingroup$
Can we combine the results of operations? For example, is $(1+3) | 36 = 436$ (where | indicates concatenation)
$endgroup$
– Hugh
Jan 27 at 22:14




$begingroup$
Can we combine the results of operations? For example, is $(1+3) | 36 = 436$ (where | indicates concatenation)
$endgroup$
– Hugh
Jan 27 at 22:14




1




1




$begingroup$
If we need to take a square root, is the two implied?
$endgroup$
– Hugh
Jan 27 at 22:14




$begingroup$
If we need to take a square root, is the two implied?
$endgroup$
– Hugh
Jan 27 at 22:14




1




1




$begingroup$
can we use factorial?
$endgroup$
– Omega Krypton
Jan 27 at 23:23




$begingroup$
can we use factorial?
$endgroup$
– Omega Krypton
Jan 27 at 23:23




3




3




$begingroup$
Factorials are most likely out, but what about parentheses, unary minus (like starting with -1) and decimal points?
$endgroup$
– Bass
Jan 28 at 0:25






$begingroup$
Factorials are most likely out, but what about parentheses, unary minus (like starting with -1) and decimal points?
$endgroup$
– Bass
Jan 28 at 0:25






1




1




$begingroup$
If decimal is allowed then round would probably valid too?
$endgroup$
– Mukyuu
Jan 28 at 3:31




$begingroup$
If decimal is allowed then round would probably valid too?
$endgroup$
– Mukyuu
Jan 28 at 3:31










8 Answers
8






active

oldest

votes


















9












$begingroup$

These get harder with larger numbers, but here are the first 40 (and a couple of the easier ones after that) with the digits in order:



1 to 10




1: $1 + 3 + 3 - 6$

2: $(1 + 3) times 3 / 6$

3: $1*3 * 3 - 6$

4: $13 - 3 - 6$

5: $-1^{33} +6$

6: $1times3-3+6$

7: $ 1 + 3 -3 +6$

8: $ 1+3/3 + 6$

9: $ 1^3 times (3+6)$

10: $ 1 + sqrt[3]3^6$




11 to 20




11: $ sqrt{1+3}+3+6$

12: $1times 3 + 3 + 6$

13: $1 + 3+3+6$

14: $-1 + 3times 3+6$

15: $-1times3 + 3times 6$

16: $1 - 3 + 3 times 6$

17: $ -1^3 +3times 6$

18: $ (1+3)*3+6 $

19: $13 + sqrt{36}$

20: $-1 + 3^3 - 6$




21 to 30




21: $ 1 * 3^3 - 6 $

22: $ 13 + 3 + 6$

23: $ -13+36 $

24: $ (1+3)timessqrt{36}$

25: $ 1 - 3 + sqrt3^6$

26: $ -1+33-6$

27: $ 1*33-6 $

28: $ 1+33-6$

29: $ -1 + 3 + sqrt3^6$

30: $ (-1+3+3)times 6$




31 to 40




31: $ 13+3*6 $

32: $ -1+3^3+6$

33: $ 13*3-6 $

34: $ 1+3^3+6$

35: $ -1+(3+3)times6 $

36: $ 1times(3+3)times 6$

37: $ 1^3+36$

38: $ sqrt{1+3}+36$

39: $ 1times3 + 36$

40: $ 1+33+6$




41 to 50 (getting much harder now, so from now on, only the easier ones)




41: $ $

42: $ (1+3+3)times 6$

43: $ $

44: $ $

45: $ 13times3+6$

46: $ $

47: $ $

48: $ (-1 + 3 times 3) times 6 $

49: $13+36$

50: $ $




51 to 60




51: $ $

52: $ $

53: $ -1 +3 times 3 times 6$

54: $ 1times 3 times 3 times 6$

55: $ 1 + 3 times 3 times 6$

56: $ $

57: $ $

58: $ (1+3)^3-6$

59: $ $

60: $ (1+3times3)times6$




61 to 70




61: $ $

62: $ $

63: $ $

64: $ (1+3/3)^6$

65: $ $

66: $ $

67: $ $

68: $ $

69: $ $

70: $ (1+3)^3+6$




71 to 80




71: $ $

72: $ (1+3)times 3 times 6$

73: $ $

74: $ $

75: $ $

76: $ $

77: $ $

78: $ 13 times sqrt{36}$

79: $ $

80: $ -1 + 3timessqrt3^6$




81 to 90




81: $ 1times3timessqrt3^6$

82: $ 1+3timessqrt3^6$

83: $ $

84: $ $

85: $ $

86: $ $

87: $ $

88: $ $

89: $ $

90: $ $




91 to 100




91: $ $

92: $ $

93: $ $

94: $ $

95: $ $

96: $ (13+3)times6 $

97: $ $

98: $ $

99: $ $

100: $ $







share|improve this answer











$endgroup$













  • $begingroup$
    Base64 because rot13 doesn't work with numbers: U29tZSBvdGhlcnM6CjQxID0gMzYgKyA2IC0gMQphbmQKNDMgPSAoM14zKSArIDE2CmFuZAo1NSA9IDYxIC0gc3FydCgzNikKYW5kCjY3ID0gNjEgLSBzcXJ0KDM2KQphbmQKNzYgPSA2MyArIDEzCmFuZAo5NCA9IDYxICsgMzM=
    $endgroup$
    – Outman
    Jan 28 at 11:19



















7












$begingroup$

Here's a solution for most of them. The remaining 21 are impossible.




  • Most of them only use simple arithmetic.

  • Some of them use exponentiation.

  • Some use square roots.


Edit: Using only the binary operators (including digit concatenation), the number of possible combinations is pretty small (arrange three binary operators, then fill in the four numbers in some order) and I double-checked all of them with a computer. Most numbers are solvable with basic arithmetic, some require exponentiation, some require negation or square roots, and the rest are apparently impossible to solve without some extra operation such as rounding.



Accordingly, it is provably impossible to construct the following twenty-one numbers:




41, 44, 46, 47, 56, 68, 69, 74, 77, 79, 83, 85, 86, 89, 90, 91, 92, 95, 97, 98, 100.




The rest can be constructed as follows:



1-25 (complete)




1. (6+1)-(3+3)

2. (3*3)-(6+1)

3. (3*3)-(6*1)

4. 13-(6+3)

5. 36-31

6. 3*(3+1)-6

7. (6+1)+(3-3)

8. (6-3)3-1

9. 6
(3-1)-3

10. 3+13-6

11. 6+3+3-1

12. (6+3+3)*1

13. 6+3+3+1

14. (6*3)-(3+1)

15. 13 + (6/3)

16. 13 + (6-3)

17. 33-16

18. (1+3)*3 + 6

19. (6 + 1/3)*3

20. (3*6)+(3-1)

21. (13-6)*3

22. 13+3+6

23. 36-13

24. (6+1)*3 +3

25. 16+(3*3)




26-50 (except 41, 44, 46, 47)




26. 33 - (6+1)

27. 33 - (6*1)

28. 61 - 33

29. 31 - (6/3)

30. (6+3+1)*3

31. 13 + (6*3)

32. 63-31

33. 13*3 - 6

34. 31+(6-3)

35. 6*(3+3) -1

36. (6+3)(3+1)

37. 6
(3+3) + 1

38. 33+(6-1)

39. 33+(6*1)

40. 31+3+6

41. round[36 + √(31)]

42. (3+3+1)*6

43. 16 + (3^3)

44. round[(3-√3)^16]

45. (3*13)+6

46. round[3 * (13 + √6)]

47. round[31√3] - 6

48. (3*3 - 1) *6

49. 36+13

50. 63-13




51-75 (except 56, 68, 69, 74)




51. 16*3 + 3

52. 61 - (3*3)

53. (3*3*6)-1

54. 1*3*3*6

55. 61-(3+3)

56. √(3136)     (!)

57. (13+6)*3

58. (3+1)^3 - 6

59. 63 - (3+1)

60. (63*1) - 3

61. 61 + (3 - 3)

62. 61 + (3/3)

63. (6+1)*(3*3)

64. 63 + (1^3)

65. 63 + (3-1)

66. 63 + (3*1)

67. 36 + 31

68. round[63 + √31]

69. round[6 √133]

70. 61+(3*3)

71. (6^3)/3 - 1

72. 36 * (3-1)

73. (6^3)/3 + 1

74. round[3 √613]

75. 13*6 - 3




76-100 (except: 77, 79, 83, 85*, 86, 89,90,91,92, 95, 97, 98, 100.)




76. 63+13

77. round[61 * (3-√3)]

78. 13 * √(36)

79. round[6*13 + √3]

80. √(3^6) * 3 - 1

81. 13*6 + 3

82. √(3^6) * 3 + 1

83. round[(3+31)*√6]

84. 3^√(16) + 3

85. -----

86. round[√(6+√3) * 31]

87. (31*3) - 6

88. 61 + (3^3)

89. round[63 * √(3-1)]

90. round[13 * √(6/3)]

91. round[16 * √33]

92. round[3^√(3*6-1)]

93. 31 * (6-3)

94. 63+31

95. round[3*31 + √6]

96. (13+3)*6

97. round[(√3)^6 * √13]

98. round[36 * (1+√3)]

99. (3*31)+6

100. round[3*(31+√6)]




If you're allowed to concatenate the results of operations (e.g. (3+1)|5 = 45 ) then a solution for 85 is :




85. (3*3)|1 - 6




Python:




from itertools import permutations
from math import sqrt, floor, ceil


concat_literal_numbers_only = True

ops = { "+" : lambda a,b: a+b,
"-" : lambda a,b: a - b,
"/" : lambda a,b : a/float(b),
"*" : lambda a,b : a*b,
"^" : lambda a,b : a**b,
"C" : lambda a,b : float(str(a) + str(b)),
"n" : lambda a : -a,
"s" : lambda a : sqrt(a),
#"f" : lambda a : floor(a)
}

arity = {"+" : 2,
"-" : 2,
"/" : 2,
"*" : 2,
"^" : 2,
"C" : 2,
"n" : 1,
"s" : 1,
"f" : 1,
}


# print ops["/"](1,3)


# args: number of open args available
# nums: available digits to be used
# ops : tuple indicating commands used so far



def evaluate(cmds) :
"""Consume the list of commands in prefix notation, producing a pair (ans, unconsumed_symbols)"""

x = cmds.pop(0)
if not ops.get(x) :
return (x, cmds)
else :
args =
for y in range(arity[x]) :
try :
(a, cmds) = evaluate(cmds)
args += [a]
except OverflowError :
return (None, None)
return (ops.get(x)(*args), cmds)

def score(ops):
ret = 0
ret += ops.count("+")
ret += 1.1*ops.count("-")
ret += 2 * ops.count("*")
ret += 3 * ops.count("/")
ret += 3 * ops.count("n")
ret += 4 * ops.count("^")
ret += 4 * ops.count("s")
ret += 4 * ops.count("f")
ret += 4 * ops.count("w")

# ret += 4 * ops.count("fs")
# ret += 4 * ops.count("cs")
return ret


agenda = [{"args" : 1, "nums" : [1,3,3,6], "ops" : }]
seen = {}


only_search_for = None
ret =

def finish(ops) :
global ret
global seen

ops_tmp = ops[:]
try :
n,_ = evaluate(ops_tmp)
except :
n = None

if n is None or not (0 score(ops) :
seen[n] = ops

print ops,"t",n




while agenda :
x = agenda.pop(0)

if not x["nums"] and not x["args"] : # finished: used up all numbers; no open spaces.
finish(x["ops"])

if len(x["nums"]) == x["args"] : # fill in numbers only
for nums in set(permutations(x["nums"])) :
finish(x["ops"] + list(nums))

# print {"args" : 0,
# "nums" : ,
# "ops" : x["ops"] + list(nums)}

elif len(x["nums"]) > x["args"] :
# add new operators




for op in ops.keys() :
if arity[op] == 1 and x["ops"] and x["ops"][-1] == op :
continue # limit repeated unary operations
if arity[op] == 1 and x["ops"] and arity.get(x["ops"][-1]) == 1 :
continue # limit repeated unary operations


if (concat_literal_numbers_only and x["ops"] and (x["ops"][-1] == "C" or (len(x["ops"])>1 and x["ops"][-2] == "C")) and op != "C") :
continue


new_x = {"args" : x["args"] + arity[op] - 1,
"nums" : x["nums"],
"ops" : x["ops"] + [op]}
agenda = [new_x] + agenda

if x["args"] == 1 :
continue

for n in set(x["nums"]) :
new_nums = x["nums"][:]
new_nums.remove(n)
new_x = {"args" : x["args"] - 1,
"nums" : new_nums,
"ops" : x["ops"] + [n]}

agenda = [new_x] + agenda

# SHOW HOW TO MAKE ALL OF THE NUMBERS
miss =
for i in range(0+1,100+1) :
if not seen.get(i) :
miss += [i]
print i, "t", seen.get(i, "---")

# SHOW WHICH NUMBERS WERE MISSED
print "missed: ", miss

# IF YOU'RE LOOKING FOR ALL POSSIBLE WAYS TO MAKE SOMETHING, SHOW THEM HERE.
if only_search_for is not None :
ret = sorted(ret, key=score)
for x in ret:
print x





share|improve this answer











$endgroup$













  • $begingroup$
    Although it's unlikely that you'd find the missing solutions there, looks like the code ignores Nth roots entirely.
    $endgroup$
    – Bass
    Jan 29 at 9:47










  • $begingroup$
    @Bass . Thanks, yes ---- when posting the code I took out some operators that didn't add to the number of solutions.
    $endgroup$
    – user326210
    Jan 30 at 1:00



















3












$begingroup$

We can generate any integer using only $1$, $3$, $3$ and $6$ with the introduction of one special function.




The function in question is the Logarithm to an arbitrary base $b$ , or $log _{b} (x)$.



To begin, let's discuss square root stacking.
$sqrt{sqrt{a}}$ is equivalent to $sqrt[4]{a}$, and $sqrt{sqrt{sqrt{a}}}$ is equivalent to $sqrt[8]{a}$, which can be rewritten as $a^frac{1}{8}$. This pattern continues indefinitely; $a$ with $n$ square roots stacked to it is equal to $a^frac{1}{2^n}$.


The laws of logarithms state that $log _{b} (x^a) = a cdot log _{b} (x)$. If we take the logarithm to base $b$ or our previous square root stack, we get $log _{b} (a^frac{1}{2^n})$, or $frac{1}{2^n} cdot log _{b}a$. Setting $a$ and $b$ as 3 means that $log _{3} (sqrt{sqrt[...]{3}})$, with $n$ square roots, is equal to $frac{1}{2^n} cdot log _{3}3$, or $2^{-n}$. $(frac{1}{a^x} = a^{-x})$

$sqrt{sqrt{16}} = 2$, and $log _{b}(b^a) = a$. As such,

$0 = -log _{sqrt{sqrt{16}}}(log_{3}(3))$
$1 = -log _{sqrt{sqrt{16}}}(log_{3}(sqrt{3}))$
$2 = -log _{sqrt{sqrt{16}}}(log_{3}(sqrt{sqrt{3}}))$
$3 = -log _{sqrt{sqrt{16}}}(log_{3}(sqrt{sqrt{sqrt{3}}}))$
$4 = -log _{sqrt{sqrt{16}}}(log_{3}(sqrt{sqrt{sqrt{sqrt{3}}}}))$

And so on, such that the amount of square roots is equal to your desired number.


This works for all integers; for negative numbers simply remove the $-$ at the start.




This was inspired by Numberphile's video on the four 4s.






share|improve this answer









$endgroup$





















    3












    $begingroup$

    Here are some:




    1: $3 + 3 - 6 + 1$

    2: $3 * 3 - (6 + 1)$

    3: $3 * 3 * 1 - 6$

    4: $3 * 3 - 6 + 1$

    5: $(3 * 6) / 3 - 1$

    6: $(3 * 6) / 3 * 1$

    7: $(3 * 6) / 3 + 1$

    8: $3 * 3 - 1 ^ 6$

    9: $(3 * 6) / (3 - 1)$

    10: $3 * 3 + 1 ^ 6$

    11: $36 / 3 - 1$

    12: $36 / 3 * 1$

    13: $36 / 3 + 1$

    14: $3 * 6 - (3 + 1)$

    15: $3 * 6 - (3 * 1)$

    16: $3 * 6 - (3 - 1)$

    17: $3 * 6 - 1 ^ 3$

    18: $3 * 6 * 1 ^ 3$

    19: $3 * 6 + 1 ^ 3$

    20: $3 * 6 + 3 - 1$

    22: (omega kyrpton did some) $3 * 6 + 3 + 1$




    I will do more later.






    share|improve this answer











    $endgroup$





















      2












      $begingroup$

      Adding some more...



      1-20: (Credits to @YoutRied)




      1: $3 + 3 - 6 + 1$

      2: $3 * 3 - (6 + 1)$

      3: $3 * 3 * 1 - 6$

      4: $3 * 3 - 6 + 1$

      5: $(3 * 6) / 3 - 1$

      6: $(3 * 6) / 3 * 1$

      7: $(3 * 6) / 3 + 1$

      8: $3 * 3 - 1 ^ 6$

      9: $(3 * 6) / (3 - 1)$

      10: $3 * 3 + 1 ^ 6$

      11: $36 / 3 - 1$

      12: $36 / 3 * 1$

      13: $36 / 3 + 1$

      14: $3 * 6 - (3 + 1)$

      15: $3 * 6 - (3 * 1)$

      16: $3 * 6 - (3 - 1)$

      17: $3 * 6 - 1 ^ 3$

      18: $3 * 6 * 1 ^ 3$

      19: $3 * 6 + 1 ^ 3$

      20: $3 * 6 + 3 - 1$




      21-29




      21: $3 * 6 + 3 * 1$

      22: $( 1 + 3 ) ! - ( 6 / 3 )$

      23: $( 1 + 3 ) ! - ( 6 - 3 )$

      24: $( 6 - 3 / 3 - 1 ) !$

      25: $1 * 3 ^ 3 - floor(sqrt{6})$

      26: $( 6 - 3 ) ^ 3 - 1$

      27: $( 6 - 3 ) ^ 3 * 1$

      28: $( 6 - 3 ) ^ 3 + 1$

      29: $31 - 6 / 3$




      41-50: (Credits to @Bass for 42, 45, 49)




      41: $ (-1+3!)+36 $

      42: $ (1+3+3)times 6$

      43: $ 31 + 6 * ceil(sqrt{3})$

      44: $floor( 1 * 3 * sqrt{6 ^ 3}) $

      45: $ 13times3+6$

      46: $ ceil(sqrt{6 ^ 3} + 31)$

      47: $ floor(sqrt{sqrt{sqrt{sqrt{sqrt{31!}}}}})+36$

      48: $6 * ( 3 * 3 - 1 )$

      49: $13+36$

      50: $ (6+1)^2 + 3 - 3$




      51-60:




      51: $( 3 * 6 - 1 ) * 3$

      52: $( 3 + 3 + 1 ) * ceil(sqrt{6})$

      53: $-1+( 3 * 3 * 6 )$

      54: $ 1*3 * 3 * 6 $

      55: $1+3*3*6$

      56: $61-3!+floor(sqrt{3})$

      57: $1*63-3!$

      58: $1+63-3!$

      59: $floor(sqrt{sqrt{sqrt{sqrt{sqrt{6^{(3-1)}}}}}}*3)$

      60: $(1+3*3)*6$




      61-70:




      61: $63-3+1$

      62: $63+1-ceil(sqrt{3})$

      63: $63-floor(sqrt{3})+1$

      64: $63+ceil(sqrt{3})-1$

      65: $63+3-1$

      66: $63+3*1$

      67: $63+3+1$

      68: $61+3!+floor(sqrt{3})$

      69: $61+3!+ceil(sqrt{3})$

      70: $61+3*3$




      71-80




      71: $(3+1)!*3-floor(sqrt{sqrt{6}})$

      72: $(3+1)*3*6$

      73: $(3+1)!*3+floor(sqrt{sqrt{6}})$

      74: $(3+1)!*3+floor(sqrt{6})$

      75: $(3+1)!*3+ceil(sqrt{6})$

      76: $ceil(sqrt{sqrt{sqrt{ceil(sqrt{sqrt{sqrt{sqrt{sqrt{ceil(sqrt{3!!})!}}}}})}}})*(3*6+1)$

      76:(much simpler) $13*6-ceil(sqrt{3})$

      77: $13*6-floor(sqrt{3})$

      78: $13*floor(sqrt{3})*6$

      79: $13*6+floor(sqrt{3})$

      80: $13*6+ceil(sqrt{3})$




      81-90:




      81: $(6+3)^{(3-1)}$

      82: $(6+3)^{ceil(sqrt{3})}+1$







      share|improve this answer











      $endgroup$













      • $begingroup$
        Who said you could use factorials?
        $endgroup$
        – Yout Ried
        Jan 28 at 0:20










      • $begingroup$
        What are number 23 (plus you probably can't use factorials and 24? I don't get them.
        $endgroup$
        – Yout Ried
        Jan 28 at 1:08












      • $begingroup$
        Oops forgot a ")" and maybe you're missing a factorial for number 24
        $endgroup$
        – Yout Ried
        Jan 28 at 1:16



















      0












      $begingroup$

      Partial answer 1-50 (w/e 41,47):




      $1= 1+3+3-6$
      $2= 1 + (frac{6}{(3+3)})$
      $3= 1^3+(frac{6}{3})$
      $4= (frac{6}{3})+3-1$
      $5= (frac{6}{3})+3^1$
      $6= 6^1+3-3$
      $7= 6+1-3+3$
      $8= 6 + 3 - 1^3$
      $9= 1^3 * (3+6)$
      $10= 1^3 +3+6$
      $11= 13 - (frac{6}{3})$
      $12= 6+3+3^1$
      $13= 6+3+3+1$
      $14= 6*3 - 3 - 1$
      $15= 6*3 - 3^1$
      $16= 16 + 3 - 3$
      $17= 16 + (frac{3}{3})$
      $18= (frac{6*3}{1^3})$
      $19= 6*3+1^3$
      $20= 6*3+3-1$
      $21= 6*3+3^1$
      $22= 6*3+3+1$
      $23= 36-13$
      $24= 6*(3+1^3)$
      $25= 16+(3*3)$
      $26= 13*(frac{6}{3})$
      $27= 33-6^1$
      $28= 33-6+1$
      $29= 31-(frac{6}{3})$
      $30= 6*(3+3-1)$
      $31= 13+3*6$
      $32= 3^3+6-1$
      $33= (frac{33}{1^6})$
      $34= 33+1^6$
      $35= (3+3)*6-1$
      $36= (3+3)^1*6$
      $37= 1+(3+3)*6$
      $38= 33+6-1$
      $39= 33+6^1$
      $40= 1+33+6$
      $41= $
      $42= (1+3+3)*6$
      $43= 16+3^3$
      $44= round(sqrt{6^3}*3^1)$
      $45= 3*3*(6-1)$
      $46= ceil(sqrt{6^3}*3)+1$
      $47= $
      $48= ((3*3)-1)*6$
      $49= 16+33$
      $50= 63-13$







      share|improve this answer











      $endgroup$





















        0












        $begingroup$

        Alrighty, I'm piggybacking off of @OmegaKrypton else and adding some of my own.



        1 to 10




        1: $1 + 3 + 3 - 6$

        2: $(1 + 3) times 3 / 6$

        3: $1^3 +3/6$

        4: $13 - 3 - 6$

        5: $-1^{33} +6$

        6: $1times3-3+6$

        7: $ 1 + 3 -3 +6$

        8: $ 1+3/3 + 6$

        9: $ 1^3 times (3+6)$

        10: $ 1^3 + 3+6$




        11 to 20




        11: $ sqrt{1+3}+3+6$

        12: $1times 3 + 3 + 6$

        13: $1 + 3+3+6$

        14: $-1 + 3times 3+6$

        15: $-1times3 + 3times 6$

        16: $1 - 3 + 3 times 6$

        17: $ -1^3 +3times 6$

        18: $ (1+3)*3+6 $

        19: $13 + sqrt{36}$

        20: $-1 + 3^3 - 6$




        21 to 30




        21: $ 1 * 3^3 - 6 $

        22: $ 13 + 3 + 6$

        23: $ -13+36 $

        24: $ (1+3)timessqrt{36}$

        25: $ 3*6+3!-1$ or $ (6-1)^(3!/3)$

        26: $ -1+33-6$

        27: $ 1*33-6 $

        28: $ 1+33-6$

        29: $ 36-3!-1$

        30: $ (-1+3+3)times 6$




        31 to 40




        31: $ 13+3*6 $

        32: $ -1+3^3+6$

        33: $ 13*3-6 $

        34: $ 1+3^3+6$

        35: $ -1+(3+3)times6 $

        36: $ 1times(3+3)times 6$

        37: $ 1^3+36$

        38: $ sqrt{1+3}+36$

        39: $ 1times3 + 36$

        40: $ 1+33+6$




        41 to 50




        41: $ $

        42: $ (1+3+3)times 6$

        43: $ 3^3+16$

        44: $ $

        45: $ 13times3+6$

        46: $ $

        47: $ $

        48: $ 16*(3!-3)$

        49: $13+36$

        50: $ 63-13$




        I added a few. It's getting late here; will come back tomorrow.






        share|improve this answer











        $endgroup$





















          0












          $begingroup$

          Expanding on Bass's answer, I added some new numbers.



          (I lost track on which numbers I added, though 1-40 is all Bass)




          1: $1 + 3 + 3 - 6$

          2: $(1 + 3) times 3 / 6$

          3: $1*3 * 3 - 6$

          4: $13 - 3 - 6$

          5: $-1^{33} +6$

          6: $1times3-3+6$

          7: $ 1 + 3 -3 +6$

          8: $ 1+3/3 + 6$

          9: $ 1^3 times (3+6)$

          10: $ 1 + sqrt[3]3^6$
          11: $ sqrt{1+3}+3+6$

          12: $1times 3 + 3 + 6$

          13: $1 + 3+3+6$

          14: $-1 + 3times 3+6$

          15: $-1times3 + 3times 6$

          16: $1 - 3 + 3 times 6$

          17: $ -1^3 +3times 6$

          18: $ (1+3)*3+6 $

          19: $13 + sqrt{36}$

          20: $-1 + 3^3 - 6$

          21: $ 1 * 3^3 - 6 $

          22: $ 13 + 3 + 6$

          23: $ -13+36 $

          24: $ (1+3)timessqrt{36}$

          25: $ 1 - 3 + sqrt3^6$

          26: $ -1+33-6$

          27: $ 1*33-6 $

          28: $ 1+33-6$

          29: $ -1 + 3 + sqrt3^6$

          30: $ (-1+3+3)times 6$

          31: $ 13+3*6 $

          32: $ -1+3^3+6$

          33: $ 13*3-6 $

          34: $ 1+3^3+6$

          35: $ -1+(3+3)times6 $

          36: $ 1times(3+3)times 6$

          37: $ 1^3+36$

          38: $ sqrt{1+3}+36$

          39: $ 1times3 + 36$

          40: $ 1+33+6$

          41: $ $

          42: $ (1+3+3)times 6$

          43: $ 3^3 + 16 $

          44: $ $

          45: $ 13times3+6$

          46: $ $

          47: $ $

          48: $ (-1 + 3 times 3) times 6 $

          49: $13+36$

          50: $ $

          51: $ 16*3+3 $

          52: $ 61 - 3times3$

          53: $ -1 +3 times 3 times 6$

          54: $ 1times 3 times 3 times 6$

          55: $ 1 + 3 times 3 times 6$

          56: $ $

          57: $ (6times3+1)times3$

          58: $ (1+3)^3-6$

          59: $ 63 - 3 - 1$

          60: $ (1+3times3)times6$

          61: $ 63 - 3 + 1$

          62: $ 63 - 1^3$

          63: $ 63 * 1^3$

          64: $ (1+3/3)^6$

          65: $ 63 + 3 - 1$

          66: $ 1 times (63 + 3) $

          67: $ 63 + 3 + 1$

          68: $ $

          69: $ $

          70: $ (1+3)^3+6 $
          71: $ 6^3 / 3 - 1 $

          72: $ (1+3)times 3 times 6$

          73: $ 6^3 / 3 + 1 $

          74: $ $

          75: $ 3^(3+1) - 6$

          76: $ 63+13 $

          77: $ $

          78: $ 13 times sqrt{36}$

          79: $ $

          80: $ -1 + 3timessqrt3^6$

          81: $ 1times3timessqrt3^6$

          82: $ 1+3timessqrt3^6$

          83: $ $

          84: $ $

          85: $ $

          86: $ $

          87: $ 3^(3+1) + 6$

          88: $ 61 + 3^3 $

          89: $ $

          90: $ $

          91: $ $

          92: $ $

          93: $ $

          94: $ 33 + 61 $

          95: $ $

          96: $ (13+3)times6 $

          97: $ $

          98: $ $

          99: $ 31times3+6$

          100: $ $




          Only need 41, 44, 46, 47, 50, 56, 68, 69, 74, 77, 79, 83, 84, 85, 86, 89, 90, 91, 92, 93, 94, 95, 97, 98, and 100 now!






          share|improve this answer










          New contributor




          Embodiment of Ignorance is a new contributor to this site. Take care in asking for clarification, commenting, and answering.
          Check out our Code of Conduct.






          $endgroup$













          • $begingroup$
            Wow that is A LOT of numbers
            $endgroup$
            – North
            Jan 30 at 3:07











          Your Answer





          StackExchange.ifUsing("editor", function () {
          return StackExchange.using("mathjaxEditing", function () {
          StackExchange.MarkdownEditor.creationCallbacks.add(function (editor, postfix) {
          StackExchange.mathjaxEditing.prepareWmdForMathJax(editor, postfix, [["$", "$"], ["\\(","\\)"]]);
          });
          });
          }, "mathjax-editing");

          StackExchange.ready(function() {
          var channelOptions = {
          tags: "".split(" "),
          id: "559"
          };
          initTagRenderer("".split(" "), "".split(" "), channelOptions);

          StackExchange.using("externalEditor", function() {
          // Have to fire editor after snippets, if snippets enabled
          if (StackExchange.settings.snippets.snippetsEnabled) {
          StackExchange.using("snippets", function() {
          createEditor();
          });
          }
          else {
          createEditor();
          }
          });

          function createEditor() {
          StackExchange.prepareEditor({
          heartbeatType: 'answer',
          autoActivateHeartbeat: false,
          convertImagesToLinks: false,
          noModals: true,
          showLowRepImageUploadWarning: true,
          reputationToPostImages: null,
          bindNavPrevention: true,
          postfix: "",
          imageUploader: {
          brandingHtml: "Powered by u003ca class="icon-imgur-white" href="https://imgur.com/"u003eu003c/au003e",
          contentPolicyHtml: "User contributions licensed under u003ca href="https://creativecommons.org/licenses/by-sa/3.0/"u003ecc by-sa 3.0 with attribution requiredu003c/au003e u003ca href="https://stackoverflow.com/legal/content-policy"u003e(content policy)u003c/au003e",
          allowUrls: true
          },
          noCode: true, onDemand: true,
          discardSelector: ".discard-answer"
          ,immediatelyShowMarkdownHelp:true
          });


          }
          });














          draft saved

          draft discarded


















          StackExchange.ready(
          function () {
          StackExchange.openid.initPostLogin('.new-post-login', 'https%3a%2f%2fpuzzling.stackexchange.com%2fquestions%2f78918%2fcreate-all-numbers-from-1-100-using-1-3-3-6%23new-answer', 'question_page');
          }
          );

          Post as a guest















          Required, but never shown

























          8 Answers
          8






          active

          oldest

          votes








          8 Answers
          8






          active

          oldest

          votes









          active

          oldest

          votes






          active

          oldest

          votes









          9












          $begingroup$

          These get harder with larger numbers, but here are the first 40 (and a couple of the easier ones after that) with the digits in order:



          1 to 10




          1: $1 + 3 + 3 - 6$

          2: $(1 + 3) times 3 / 6$

          3: $1*3 * 3 - 6$

          4: $13 - 3 - 6$

          5: $-1^{33} +6$

          6: $1times3-3+6$

          7: $ 1 + 3 -3 +6$

          8: $ 1+3/3 + 6$

          9: $ 1^3 times (3+6)$

          10: $ 1 + sqrt[3]3^6$




          11 to 20




          11: $ sqrt{1+3}+3+6$

          12: $1times 3 + 3 + 6$

          13: $1 + 3+3+6$

          14: $-1 + 3times 3+6$

          15: $-1times3 + 3times 6$

          16: $1 - 3 + 3 times 6$

          17: $ -1^3 +3times 6$

          18: $ (1+3)*3+6 $

          19: $13 + sqrt{36}$

          20: $-1 + 3^3 - 6$




          21 to 30




          21: $ 1 * 3^3 - 6 $

          22: $ 13 + 3 + 6$

          23: $ -13+36 $

          24: $ (1+3)timessqrt{36}$

          25: $ 1 - 3 + sqrt3^6$

          26: $ -1+33-6$

          27: $ 1*33-6 $

          28: $ 1+33-6$

          29: $ -1 + 3 + sqrt3^6$

          30: $ (-1+3+3)times 6$




          31 to 40




          31: $ 13+3*6 $

          32: $ -1+3^3+6$

          33: $ 13*3-6 $

          34: $ 1+3^3+6$

          35: $ -1+(3+3)times6 $

          36: $ 1times(3+3)times 6$

          37: $ 1^3+36$

          38: $ sqrt{1+3}+36$

          39: $ 1times3 + 36$

          40: $ 1+33+6$




          41 to 50 (getting much harder now, so from now on, only the easier ones)




          41: $ $

          42: $ (1+3+3)times 6$

          43: $ $

          44: $ $

          45: $ 13times3+6$

          46: $ $

          47: $ $

          48: $ (-1 + 3 times 3) times 6 $

          49: $13+36$

          50: $ $




          51 to 60




          51: $ $

          52: $ $

          53: $ -1 +3 times 3 times 6$

          54: $ 1times 3 times 3 times 6$

          55: $ 1 + 3 times 3 times 6$

          56: $ $

          57: $ $

          58: $ (1+3)^3-6$

          59: $ $

          60: $ (1+3times3)times6$




          61 to 70




          61: $ $

          62: $ $

          63: $ $

          64: $ (1+3/3)^6$

          65: $ $

          66: $ $

          67: $ $

          68: $ $

          69: $ $

          70: $ (1+3)^3+6$




          71 to 80




          71: $ $

          72: $ (1+3)times 3 times 6$

          73: $ $

          74: $ $

          75: $ $

          76: $ $

          77: $ $

          78: $ 13 times sqrt{36}$

          79: $ $

          80: $ -1 + 3timessqrt3^6$




          81 to 90




          81: $ 1times3timessqrt3^6$

          82: $ 1+3timessqrt3^6$

          83: $ $

          84: $ $

          85: $ $

          86: $ $

          87: $ $

          88: $ $

          89: $ $

          90: $ $




          91 to 100




          91: $ $

          92: $ $

          93: $ $

          94: $ $

          95: $ $

          96: $ (13+3)times6 $

          97: $ $

          98: $ $

          99: $ $

          100: $ $







          share|improve this answer











          $endgroup$













          • $begingroup$
            Base64 because rot13 doesn't work with numbers: U29tZSBvdGhlcnM6CjQxID0gMzYgKyA2IC0gMQphbmQKNDMgPSAoM14zKSArIDE2CmFuZAo1NSA9IDYxIC0gc3FydCgzNikKYW5kCjY3ID0gNjEgLSBzcXJ0KDM2KQphbmQKNzYgPSA2MyArIDEzCmFuZAo5NCA9IDYxICsgMzM=
            $endgroup$
            – Outman
            Jan 28 at 11:19
















          9












          $begingroup$

          These get harder with larger numbers, but here are the first 40 (and a couple of the easier ones after that) with the digits in order:



          1 to 10




          1: $1 + 3 + 3 - 6$

          2: $(1 + 3) times 3 / 6$

          3: $1*3 * 3 - 6$

          4: $13 - 3 - 6$

          5: $-1^{33} +6$

          6: $1times3-3+6$

          7: $ 1 + 3 -3 +6$

          8: $ 1+3/3 + 6$

          9: $ 1^3 times (3+6)$

          10: $ 1 + sqrt[3]3^6$




          11 to 20




          11: $ sqrt{1+3}+3+6$

          12: $1times 3 + 3 + 6$

          13: $1 + 3+3+6$

          14: $-1 + 3times 3+6$

          15: $-1times3 + 3times 6$

          16: $1 - 3 + 3 times 6$

          17: $ -1^3 +3times 6$

          18: $ (1+3)*3+6 $

          19: $13 + sqrt{36}$

          20: $-1 + 3^3 - 6$




          21 to 30




          21: $ 1 * 3^3 - 6 $

          22: $ 13 + 3 + 6$

          23: $ -13+36 $

          24: $ (1+3)timessqrt{36}$

          25: $ 1 - 3 + sqrt3^6$

          26: $ -1+33-6$

          27: $ 1*33-6 $

          28: $ 1+33-6$

          29: $ -1 + 3 + sqrt3^6$

          30: $ (-1+3+3)times 6$




          31 to 40




          31: $ 13+3*6 $

          32: $ -1+3^3+6$

          33: $ 13*3-6 $

          34: $ 1+3^3+6$

          35: $ -1+(3+3)times6 $

          36: $ 1times(3+3)times 6$

          37: $ 1^3+36$

          38: $ sqrt{1+3}+36$

          39: $ 1times3 + 36$

          40: $ 1+33+6$




          41 to 50 (getting much harder now, so from now on, only the easier ones)




          41: $ $

          42: $ (1+3+3)times 6$

          43: $ $

          44: $ $

          45: $ 13times3+6$

          46: $ $

          47: $ $

          48: $ (-1 + 3 times 3) times 6 $

          49: $13+36$

          50: $ $




          51 to 60




          51: $ $

          52: $ $

          53: $ -1 +3 times 3 times 6$

          54: $ 1times 3 times 3 times 6$

          55: $ 1 + 3 times 3 times 6$

          56: $ $

          57: $ $

          58: $ (1+3)^3-6$

          59: $ $

          60: $ (1+3times3)times6$




          61 to 70




          61: $ $

          62: $ $

          63: $ $

          64: $ (1+3/3)^6$

          65: $ $

          66: $ $

          67: $ $

          68: $ $

          69: $ $

          70: $ (1+3)^3+6$




          71 to 80




          71: $ $

          72: $ (1+3)times 3 times 6$

          73: $ $

          74: $ $

          75: $ $

          76: $ $

          77: $ $

          78: $ 13 times sqrt{36}$

          79: $ $

          80: $ -1 + 3timessqrt3^6$




          81 to 90




          81: $ 1times3timessqrt3^6$

          82: $ 1+3timessqrt3^6$

          83: $ $

          84: $ $

          85: $ $

          86: $ $

          87: $ $

          88: $ $

          89: $ $

          90: $ $




          91 to 100




          91: $ $

          92: $ $

          93: $ $

          94: $ $

          95: $ $

          96: $ (13+3)times6 $

          97: $ $

          98: $ $

          99: $ $

          100: $ $







          share|improve this answer











          $endgroup$













          • $begingroup$
            Base64 because rot13 doesn't work with numbers: U29tZSBvdGhlcnM6CjQxID0gMzYgKyA2IC0gMQphbmQKNDMgPSAoM14zKSArIDE2CmFuZAo1NSA9IDYxIC0gc3FydCgzNikKYW5kCjY3ID0gNjEgLSBzcXJ0KDM2KQphbmQKNzYgPSA2MyArIDEzCmFuZAo5NCA9IDYxICsgMzM=
            $endgroup$
            – Outman
            Jan 28 at 11:19














          9












          9








          9





          $begingroup$

          These get harder with larger numbers, but here are the first 40 (and a couple of the easier ones after that) with the digits in order:



          1 to 10




          1: $1 + 3 + 3 - 6$

          2: $(1 + 3) times 3 / 6$

          3: $1*3 * 3 - 6$

          4: $13 - 3 - 6$

          5: $-1^{33} +6$

          6: $1times3-3+6$

          7: $ 1 + 3 -3 +6$

          8: $ 1+3/3 + 6$

          9: $ 1^3 times (3+6)$

          10: $ 1 + sqrt[3]3^6$




          11 to 20




          11: $ sqrt{1+3}+3+6$

          12: $1times 3 + 3 + 6$

          13: $1 + 3+3+6$

          14: $-1 + 3times 3+6$

          15: $-1times3 + 3times 6$

          16: $1 - 3 + 3 times 6$

          17: $ -1^3 +3times 6$

          18: $ (1+3)*3+6 $

          19: $13 + sqrt{36}$

          20: $-1 + 3^3 - 6$




          21 to 30




          21: $ 1 * 3^3 - 6 $

          22: $ 13 + 3 + 6$

          23: $ -13+36 $

          24: $ (1+3)timessqrt{36}$

          25: $ 1 - 3 + sqrt3^6$

          26: $ -1+33-6$

          27: $ 1*33-6 $

          28: $ 1+33-6$

          29: $ -1 + 3 + sqrt3^6$

          30: $ (-1+3+3)times 6$




          31 to 40




          31: $ 13+3*6 $

          32: $ -1+3^3+6$

          33: $ 13*3-6 $

          34: $ 1+3^3+6$

          35: $ -1+(3+3)times6 $

          36: $ 1times(3+3)times 6$

          37: $ 1^3+36$

          38: $ sqrt{1+3}+36$

          39: $ 1times3 + 36$

          40: $ 1+33+6$




          41 to 50 (getting much harder now, so from now on, only the easier ones)




          41: $ $

          42: $ (1+3+3)times 6$

          43: $ $

          44: $ $

          45: $ 13times3+6$

          46: $ $

          47: $ $

          48: $ (-1 + 3 times 3) times 6 $

          49: $13+36$

          50: $ $




          51 to 60




          51: $ $

          52: $ $

          53: $ -1 +3 times 3 times 6$

          54: $ 1times 3 times 3 times 6$

          55: $ 1 + 3 times 3 times 6$

          56: $ $

          57: $ $

          58: $ (1+3)^3-6$

          59: $ $

          60: $ (1+3times3)times6$




          61 to 70




          61: $ $

          62: $ $

          63: $ $

          64: $ (1+3/3)^6$

          65: $ $

          66: $ $

          67: $ $

          68: $ $

          69: $ $

          70: $ (1+3)^3+6$




          71 to 80




          71: $ $

          72: $ (1+3)times 3 times 6$

          73: $ $

          74: $ $

          75: $ $

          76: $ $

          77: $ $

          78: $ 13 times sqrt{36}$

          79: $ $

          80: $ -1 + 3timessqrt3^6$




          81 to 90




          81: $ 1times3timessqrt3^6$

          82: $ 1+3timessqrt3^6$

          83: $ $

          84: $ $

          85: $ $

          86: $ $

          87: $ $

          88: $ $

          89: $ $

          90: $ $




          91 to 100




          91: $ $

          92: $ $

          93: $ $

          94: $ $

          95: $ $

          96: $ (13+3)times6 $

          97: $ $

          98: $ $

          99: $ $

          100: $ $







          share|improve this answer











          $endgroup$



          These get harder with larger numbers, but here are the first 40 (and a couple of the easier ones after that) with the digits in order:



          1 to 10




          1: $1 + 3 + 3 - 6$

          2: $(1 + 3) times 3 / 6$

          3: $1*3 * 3 - 6$

          4: $13 - 3 - 6$

          5: $-1^{33} +6$

          6: $1times3-3+6$

          7: $ 1 + 3 -3 +6$

          8: $ 1+3/3 + 6$

          9: $ 1^3 times (3+6)$

          10: $ 1 + sqrt[3]3^6$




          11 to 20




          11: $ sqrt{1+3}+3+6$

          12: $1times 3 + 3 + 6$

          13: $1 + 3+3+6$

          14: $-1 + 3times 3+6$

          15: $-1times3 + 3times 6$

          16: $1 - 3 + 3 times 6$

          17: $ -1^3 +3times 6$

          18: $ (1+3)*3+6 $

          19: $13 + sqrt{36}$

          20: $-1 + 3^3 - 6$




          21 to 30




          21: $ 1 * 3^3 - 6 $

          22: $ 13 + 3 + 6$

          23: $ -13+36 $

          24: $ (1+3)timessqrt{36}$

          25: $ 1 - 3 + sqrt3^6$

          26: $ -1+33-6$

          27: $ 1*33-6 $

          28: $ 1+33-6$

          29: $ -1 + 3 + sqrt3^6$

          30: $ (-1+3+3)times 6$




          31 to 40




          31: $ 13+3*6 $

          32: $ -1+3^3+6$

          33: $ 13*3-6 $

          34: $ 1+3^3+6$

          35: $ -1+(3+3)times6 $

          36: $ 1times(3+3)times 6$

          37: $ 1^3+36$

          38: $ sqrt{1+3}+36$

          39: $ 1times3 + 36$

          40: $ 1+33+6$




          41 to 50 (getting much harder now, so from now on, only the easier ones)




          41: $ $

          42: $ (1+3+3)times 6$

          43: $ $

          44: $ $

          45: $ 13times3+6$

          46: $ $

          47: $ $

          48: $ (-1 + 3 times 3) times 6 $

          49: $13+36$

          50: $ $




          51 to 60




          51: $ $

          52: $ $

          53: $ -1 +3 times 3 times 6$

          54: $ 1times 3 times 3 times 6$

          55: $ 1 + 3 times 3 times 6$

          56: $ $

          57: $ $

          58: $ (1+3)^3-6$

          59: $ $

          60: $ (1+3times3)times6$




          61 to 70




          61: $ $

          62: $ $

          63: $ $

          64: $ (1+3/3)^6$

          65: $ $

          66: $ $

          67: $ $

          68: $ $

          69: $ $

          70: $ (1+3)^3+6$




          71 to 80




          71: $ $

          72: $ (1+3)times 3 times 6$

          73: $ $

          74: $ $

          75: $ $

          76: $ $

          77: $ $

          78: $ 13 times sqrt{36}$

          79: $ $

          80: $ -1 + 3timessqrt3^6$




          81 to 90




          81: $ 1times3timessqrt3^6$

          82: $ 1+3timessqrt3^6$

          83: $ $

          84: $ $

          85: $ $

          86: $ $

          87: $ $

          88: $ $

          89: $ $

          90: $ $




          91 to 100




          91: $ $

          92: $ $

          93: $ $

          94: $ $

          95: $ $

          96: $ (13+3)times6 $

          97: $ $

          98: $ $

          99: $ $

          100: $ $








          share|improve this answer














          share|improve this answer



          share|improve this answer








          edited Jan 28 at 13:29

























          answered Jan 28 at 1:17









          BassBass

          29.2k470178




          29.2k470178












          • $begingroup$
            Base64 because rot13 doesn't work with numbers: U29tZSBvdGhlcnM6CjQxID0gMzYgKyA2IC0gMQphbmQKNDMgPSAoM14zKSArIDE2CmFuZAo1NSA9IDYxIC0gc3FydCgzNikKYW5kCjY3ID0gNjEgLSBzcXJ0KDM2KQphbmQKNzYgPSA2MyArIDEzCmFuZAo5NCA9IDYxICsgMzM=
            $endgroup$
            – Outman
            Jan 28 at 11:19


















          • $begingroup$
            Base64 because rot13 doesn't work with numbers: U29tZSBvdGhlcnM6CjQxID0gMzYgKyA2IC0gMQphbmQKNDMgPSAoM14zKSArIDE2CmFuZAo1NSA9IDYxIC0gc3FydCgzNikKYW5kCjY3ID0gNjEgLSBzcXJ0KDM2KQphbmQKNzYgPSA2MyArIDEzCmFuZAo5NCA9IDYxICsgMzM=
            $endgroup$
            – Outman
            Jan 28 at 11:19
















          $begingroup$
          Base64 because rot13 doesn't work with numbers: U29tZSBvdGhlcnM6CjQxID0gMzYgKyA2IC0gMQphbmQKNDMgPSAoM14zKSArIDE2CmFuZAo1NSA9IDYxIC0gc3FydCgzNikKYW5kCjY3ID0gNjEgLSBzcXJ0KDM2KQphbmQKNzYgPSA2MyArIDEzCmFuZAo5NCA9IDYxICsgMzM=
          $endgroup$
          – Outman
          Jan 28 at 11:19




          $begingroup$
          Base64 because rot13 doesn't work with numbers: U29tZSBvdGhlcnM6CjQxID0gMzYgKyA2IC0gMQphbmQKNDMgPSAoM14zKSArIDE2CmFuZAo1NSA9IDYxIC0gc3FydCgzNikKYW5kCjY3ID0gNjEgLSBzcXJ0KDM2KQphbmQKNzYgPSA2MyArIDEzCmFuZAo5NCA9IDYxICsgMzM=
          $endgroup$
          – Outman
          Jan 28 at 11:19











          7












          $begingroup$

          Here's a solution for most of them. The remaining 21 are impossible.




          • Most of them only use simple arithmetic.

          • Some of them use exponentiation.

          • Some use square roots.


          Edit: Using only the binary operators (including digit concatenation), the number of possible combinations is pretty small (arrange three binary operators, then fill in the four numbers in some order) and I double-checked all of them with a computer. Most numbers are solvable with basic arithmetic, some require exponentiation, some require negation or square roots, and the rest are apparently impossible to solve without some extra operation such as rounding.



          Accordingly, it is provably impossible to construct the following twenty-one numbers:




          41, 44, 46, 47, 56, 68, 69, 74, 77, 79, 83, 85, 86, 89, 90, 91, 92, 95, 97, 98, 100.




          The rest can be constructed as follows:



          1-25 (complete)




          1. (6+1)-(3+3)

          2. (3*3)-(6+1)

          3. (3*3)-(6*1)

          4. 13-(6+3)

          5. 36-31

          6. 3*(3+1)-6

          7. (6+1)+(3-3)

          8. (6-3)3-1

          9. 6
          (3-1)-3

          10. 3+13-6

          11. 6+3+3-1

          12. (6+3+3)*1

          13. 6+3+3+1

          14. (6*3)-(3+1)

          15. 13 + (6/3)

          16. 13 + (6-3)

          17. 33-16

          18. (1+3)*3 + 6

          19. (6 + 1/3)*3

          20. (3*6)+(3-1)

          21. (13-6)*3

          22. 13+3+6

          23. 36-13

          24. (6+1)*3 +3

          25. 16+(3*3)




          26-50 (except 41, 44, 46, 47)




          26. 33 - (6+1)

          27. 33 - (6*1)

          28. 61 - 33

          29. 31 - (6/3)

          30. (6+3+1)*3

          31. 13 + (6*3)

          32. 63-31

          33. 13*3 - 6

          34. 31+(6-3)

          35. 6*(3+3) -1

          36. (6+3)(3+1)

          37. 6
          (3+3) + 1

          38. 33+(6-1)

          39. 33+(6*1)

          40. 31+3+6

          41. round[36 + √(31)]

          42. (3+3+1)*6

          43. 16 + (3^3)

          44. round[(3-√3)^16]

          45. (3*13)+6

          46. round[3 * (13 + √6)]

          47. round[31√3] - 6

          48. (3*3 - 1) *6

          49. 36+13

          50. 63-13




          51-75 (except 56, 68, 69, 74)




          51. 16*3 + 3

          52. 61 - (3*3)

          53. (3*3*6)-1

          54. 1*3*3*6

          55. 61-(3+3)

          56. √(3136)     (!)

          57. (13+6)*3

          58. (3+1)^3 - 6

          59. 63 - (3+1)

          60. (63*1) - 3

          61. 61 + (3 - 3)

          62. 61 + (3/3)

          63. (6+1)*(3*3)

          64. 63 + (1^3)

          65. 63 + (3-1)

          66. 63 + (3*1)

          67. 36 + 31

          68. round[63 + √31]

          69. round[6 √133]

          70. 61+(3*3)

          71. (6^3)/3 - 1

          72. 36 * (3-1)

          73. (6^3)/3 + 1

          74. round[3 √613]

          75. 13*6 - 3




          76-100 (except: 77, 79, 83, 85*, 86, 89,90,91,92, 95, 97, 98, 100.)




          76. 63+13

          77. round[61 * (3-√3)]

          78. 13 * √(36)

          79. round[6*13 + √3]

          80. √(3^6) * 3 - 1

          81. 13*6 + 3

          82. √(3^6) * 3 + 1

          83. round[(3+31)*√6]

          84. 3^√(16) + 3

          85. -----

          86. round[√(6+√3) * 31]

          87. (31*3) - 6

          88. 61 + (3^3)

          89. round[63 * √(3-1)]

          90. round[13 * √(6/3)]

          91. round[16 * √33]

          92. round[3^√(3*6-1)]

          93. 31 * (6-3)

          94. 63+31

          95. round[3*31 + √6]

          96. (13+3)*6

          97. round[(√3)^6 * √13]

          98. round[36 * (1+√3)]

          99. (3*31)+6

          100. round[3*(31+√6)]




          If you're allowed to concatenate the results of operations (e.g. (3+1)|5 = 45 ) then a solution for 85 is :




          85. (3*3)|1 - 6




          Python:




          from itertools import permutations
          from math import sqrt, floor, ceil


          concat_literal_numbers_only = True

          ops = { "+" : lambda a,b: a+b,
          "-" : lambda a,b: a - b,
          "/" : lambda a,b : a/float(b),
          "*" : lambda a,b : a*b,
          "^" : lambda a,b : a**b,
          "C" : lambda a,b : float(str(a) + str(b)),
          "n" : lambda a : -a,
          "s" : lambda a : sqrt(a),
          #"f" : lambda a : floor(a)
          }

          arity = {"+" : 2,
          "-" : 2,
          "/" : 2,
          "*" : 2,
          "^" : 2,
          "C" : 2,
          "n" : 1,
          "s" : 1,
          "f" : 1,
          }


          # print ops["/"](1,3)


          # args: number of open args available
          # nums: available digits to be used
          # ops : tuple indicating commands used so far



          def evaluate(cmds) :
          """Consume the list of commands in prefix notation, producing a pair (ans, unconsumed_symbols)"""

          x = cmds.pop(0)
          if not ops.get(x) :
          return (x, cmds)
          else :
          args =
          for y in range(arity[x]) :
          try :
          (a, cmds) = evaluate(cmds)
          args += [a]
          except OverflowError :
          return (None, None)
          return (ops.get(x)(*args), cmds)

          def score(ops):
          ret = 0
          ret += ops.count("+")
          ret += 1.1*ops.count("-")
          ret += 2 * ops.count("*")
          ret += 3 * ops.count("/")
          ret += 3 * ops.count("n")
          ret += 4 * ops.count("^")
          ret += 4 * ops.count("s")
          ret += 4 * ops.count("f")
          ret += 4 * ops.count("w")

          # ret += 4 * ops.count("fs")
          # ret += 4 * ops.count("cs")
          return ret


          agenda = [{"args" : 1, "nums" : [1,3,3,6], "ops" : }]
          seen = {}


          only_search_for = None
          ret =

          def finish(ops) :
          global ret
          global seen

          ops_tmp = ops[:]
          try :
          n,_ = evaluate(ops_tmp)
          except :
          n = None

          if n is None or not (0 score(ops) :
          seen[n] = ops

          print ops,"t",n




          while agenda :
          x = agenda.pop(0)

          if not x["nums"] and not x["args"] : # finished: used up all numbers; no open spaces.
          finish(x["ops"])

          if len(x["nums"]) == x["args"] : # fill in numbers only
          for nums in set(permutations(x["nums"])) :
          finish(x["ops"] + list(nums))

          # print {"args" : 0,
          # "nums" : ,
          # "ops" : x["ops"] + list(nums)}

          elif len(x["nums"]) > x["args"] :
          # add new operators




          for op in ops.keys() :
          if arity[op] == 1 and x["ops"] and x["ops"][-1] == op :
          continue # limit repeated unary operations
          if arity[op] == 1 and x["ops"] and arity.get(x["ops"][-1]) == 1 :
          continue # limit repeated unary operations


          if (concat_literal_numbers_only and x["ops"] and (x["ops"][-1] == "C" or (len(x["ops"])>1 and x["ops"][-2] == "C")) and op != "C") :
          continue


          new_x = {"args" : x["args"] + arity[op] - 1,
          "nums" : x["nums"],
          "ops" : x["ops"] + [op]}
          agenda = [new_x] + agenda

          if x["args"] == 1 :
          continue

          for n in set(x["nums"]) :
          new_nums = x["nums"][:]
          new_nums.remove(n)
          new_x = {"args" : x["args"] - 1,
          "nums" : new_nums,
          "ops" : x["ops"] + [n]}

          agenda = [new_x] + agenda

          # SHOW HOW TO MAKE ALL OF THE NUMBERS
          miss =
          for i in range(0+1,100+1) :
          if not seen.get(i) :
          miss += [i]
          print i, "t", seen.get(i, "---")

          # SHOW WHICH NUMBERS WERE MISSED
          print "missed: ", miss

          # IF YOU'RE LOOKING FOR ALL POSSIBLE WAYS TO MAKE SOMETHING, SHOW THEM HERE.
          if only_search_for is not None :
          ret = sorted(ret, key=score)
          for x in ret:
          print x





          share|improve this answer











          $endgroup$













          • $begingroup$
            Although it's unlikely that you'd find the missing solutions there, looks like the code ignores Nth roots entirely.
            $endgroup$
            – Bass
            Jan 29 at 9:47










          • $begingroup$
            @Bass . Thanks, yes ---- when posting the code I took out some operators that didn't add to the number of solutions.
            $endgroup$
            – user326210
            Jan 30 at 1:00
















          7












          $begingroup$

          Here's a solution for most of them. The remaining 21 are impossible.




          • Most of them only use simple arithmetic.

          • Some of them use exponentiation.

          • Some use square roots.


          Edit: Using only the binary operators (including digit concatenation), the number of possible combinations is pretty small (arrange three binary operators, then fill in the four numbers in some order) and I double-checked all of them with a computer. Most numbers are solvable with basic arithmetic, some require exponentiation, some require negation or square roots, and the rest are apparently impossible to solve without some extra operation such as rounding.



          Accordingly, it is provably impossible to construct the following twenty-one numbers:




          41, 44, 46, 47, 56, 68, 69, 74, 77, 79, 83, 85, 86, 89, 90, 91, 92, 95, 97, 98, 100.




          The rest can be constructed as follows:



          1-25 (complete)




          1. (6+1)-(3+3)

          2. (3*3)-(6+1)

          3. (3*3)-(6*1)

          4. 13-(6+3)

          5. 36-31

          6. 3*(3+1)-6

          7. (6+1)+(3-3)

          8. (6-3)3-1

          9. 6
          (3-1)-3

          10. 3+13-6

          11. 6+3+3-1

          12. (6+3+3)*1

          13. 6+3+3+1

          14. (6*3)-(3+1)

          15. 13 + (6/3)

          16. 13 + (6-3)

          17. 33-16

          18. (1+3)*3 + 6

          19. (6 + 1/3)*3

          20. (3*6)+(3-1)

          21. (13-6)*3

          22. 13+3+6

          23. 36-13

          24. (6+1)*3 +3

          25. 16+(3*3)




          26-50 (except 41, 44, 46, 47)




          26. 33 - (6+1)

          27. 33 - (6*1)

          28. 61 - 33

          29. 31 - (6/3)

          30. (6+3+1)*3

          31. 13 + (6*3)

          32. 63-31

          33. 13*3 - 6

          34. 31+(6-3)

          35. 6*(3+3) -1

          36. (6+3)(3+1)

          37. 6
          (3+3) + 1

          38. 33+(6-1)

          39. 33+(6*1)

          40. 31+3+6

          41. round[36 + √(31)]

          42. (3+3+1)*6

          43. 16 + (3^3)

          44. round[(3-√3)^16]

          45. (3*13)+6

          46. round[3 * (13 + √6)]

          47. round[31√3] - 6

          48. (3*3 - 1) *6

          49. 36+13

          50. 63-13




          51-75 (except 56, 68, 69, 74)




          51. 16*3 + 3

          52. 61 - (3*3)

          53. (3*3*6)-1

          54. 1*3*3*6

          55. 61-(3+3)

          56. √(3136)     (!)

          57. (13+6)*3

          58. (3+1)^3 - 6

          59. 63 - (3+1)

          60. (63*1) - 3

          61. 61 + (3 - 3)

          62. 61 + (3/3)

          63. (6+1)*(3*3)

          64. 63 + (1^3)

          65. 63 + (3-1)

          66. 63 + (3*1)

          67. 36 + 31

          68. round[63 + √31]

          69. round[6 √133]

          70. 61+(3*3)

          71. (6^3)/3 - 1

          72. 36 * (3-1)

          73. (6^3)/3 + 1

          74. round[3 √613]

          75. 13*6 - 3




          76-100 (except: 77, 79, 83, 85*, 86, 89,90,91,92, 95, 97, 98, 100.)




          76. 63+13

          77. round[61 * (3-√3)]

          78. 13 * √(36)

          79. round[6*13 + √3]

          80. √(3^6) * 3 - 1

          81. 13*6 + 3

          82. √(3^6) * 3 + 1

          83. round[(3+31)*√6]

          84. 3^√(16) + 3

          85. -----

          86. round[√(6+√3) * 31]

          87. (31*3) - 6

          88. 61 + (3^3)

          89. round[63 * √(3-1)]

          90. round[13 * √(6/3)]

          91. round[16 * √33]

          92. round[3^√(3*6-1)]

          93. 31 * (6-3)

          94. 63+31

          95. round[3*31 + √6]

          96. (13+3)*6

          97. round[(√3)^6 * √13]

          98. round[36 * (1+√3)]

          99. (3*31)+6

          100. round[3*(31+√6)]




          If you're allowed to concatenate the results of operations (e.g. (3+1)|5 = 45 ) then a solution for 85 is :




          85. (3*3)|1 - 6




          Python:




          from itertools import permutations
          from math import sqrt, floor, ceil


          concat_literal_numbers_only = True

          ops = { "+" : lambda a,b: a+b,
          "-" : lambda a,b: a - b,
          "/" : lambda a,b : a/float(b),
          "*" : lambda a,b : a*b,
          "^" : lambda a,b : a**b,
          "C" : lambda a,b : float(str(a) + str(b)),
          "n" : lambda a : -a,
          "s" : lambda a : sqrt(a),
          #"f" : lambda a : floor(a)
          }

          arity = {"+" : 2,
          "-" : 2,
          "/" : 2,
          "*" : 2,
          "^" : 2,
          "C" : 2,
          "n" : 1,
          "s" : 1,
          "f" : 1,
          }


          # print ops["/"](1,3)


          # args: number of open args available
          # nums: available digits to be used
          # ops : tuple indicating commands used so far



          def evaluate(cmds) :
          """Consume the list of commands in prefix notation, producing a pair (ans, unconsumed_symbols)"""

          x = cmds.pop(0)
          if not ops.get(x) :
          return (x, cmds)
          else :
          args =
          for y in range(arity[x]) :
          try :
          (a, cmds) = evaluate(cmds)
          args += [a]
          except OverflowError :
          return (None, None)
          return (ops.get(x)(*args), cmds)

          def score(ops):
          ret = 0
          ret += ops.count("+")
          ret += 1.1*ops.count("-")
          ret += 2 * ops.count("*")
          ret += 3 * ops.count("/")
          ret += 3 * ops.count("n")
          ret += 4 * ops.count("^")
          ret += 4 * ops.count("s")
          ret += 4 * ops.count("f")
          ret += 4 * ops.count("w")

          # ret += 4 * ops.count("fs")
          # ret += 4 * ops.count("cs")
          return ret


          agenda = [{"args" : 1, "nums" : [1,3,3,6], "ops" : }]
          seen = {}


          only_search_for = None
          ret =

          def finish(ops) :
          global ret
          global seen

          ops_tmp = ops[:]
          try :
          n,_ = evaluate(ops_tmp)
          except :
          n = None

          if n is None or not (0 score(ops) :
          seen[n] = ops

          print ops,"t",n




          while agenda :
          x = agenda.pop(0)

          if not x["nums"] and not x["args"] : # finished: used up all numbers; no open spaces.
          finish(x["ops"])

          if len(x["nums"]) == x["args"] : # fill in numbers only
          for nums in set(permutations(x["nums"])) :
          finish(x["ops"] + list(nums))

          # print {"args" : 0,
          # "nums" : ,
          # "ops" : x["ops"] + list(nums)}

          elif len(x["nums"]) > x["args"] :
          # add new operators




          for op in ops.keys() :
          if arity[op] == 1 and x["ops"] and x["ops"][-1] == op :
          continue # limit repeated unary operations
          if arity[op] == 1 and x["ops"] and arity.get(x["ops"][-1]) == 1 :
          continue # limit repeated unary operations


          if (concat_literal_numbers_only and x["ops"] and (x["ops"][-1] == "C" or (len(x["ops"])>1 and x["ops"][-2] == "C")) and op != "C") :
          continue


          new_x = {"args" : x["args"] + arity[op] - 1,
          "nums" : x["nums"],
          "ops" : x["ops"] + [op]}
          agenda = [new_x] + agenda

          if x["args"] == 1 :
          continue

          for n in set(x["nums"]) :
          new_nums = x["nums"][:]
          new_nums.remove(n)
          new_x = {"args" : x["args"] - 1,
          "nums" : new_nums,
          "ops" : x["ops"] + [n]}

          agenda = [new_x] + agenda

          # SHOW HOW TO MAKE ALL OF THE NUMBERS
          miss =
          for i in range(0+1,100+1) :
          if not seen.get(i) :
          miss += [i]
          print i, "t", seen.get(i, "---")

          # SHOW WHICH NUMBERS WERE MISSED
          print "missed: ", miss

          # IF YOU'RE LOOKING FOR ALL POSSIBLE WAYS TO MAKE SOMETHING, SHOW THEM HERE.
          if only_search_for is not None :
          ret = sorted(ret, key=score)
          for x in ret:
          print x





          share|improve this answer











          $endgroup$













          • $begingroup$
            Although it's unlikely that you'd find the missing solutions there, looks like the code ignores Nth roots entirely.
            $endgroup$
            – Bass
            Jan 29 at 9:47










          • $begingroup$
            @Bass . Thanks, yes ---- when posting the code I took out some operators that didn't add to the number of solutions.
            $endgroup$
            – user326210
            Jan 30 at 1:00














          7












          7








          7





          $begingroup$

          Here's a solution for most of them. The remaining 21 are impossible.




          • Most of them only use simple arithmetic.

          • Some of them use exponentiation.

          • Some use square roots.


          Edit: Using only the binary operators (including digit concatenation), the number of possible combinations is pretty small (arrange three binary operators, then fill in the four numbers in some order) and I double-checked all of them with a computer. Most numbers are solvable with basic arithmetic, some require exponentiation, some require negation or square roots, and the rest are apparently impossible to solve without some extra operation such as rounding.



          Accordingly, it is provably impossible to construct the following twenty-one numbers:




          41, 44, 46, 47, 56, 68, 69, 74, 77, 79, 83, 85, 86, 89, 90, 91, 92, 95, 97, 98, 100.




          The rest can be constructed as follows:



          1-25 (complete)




          1. (6+1)-(3+3)

          2. (3*3)-(6+1)

          3. (3*3)-(6*1)

          4. 13-(6+3)

          5. 36-31

          6. 3*(3+1)-6

          7. (6+1)+(3-3)

          8. (6-3)3-1

          9. 6
          (3-1)-3

          10. 3+13-6

          11. 6+3+3-1

          12. (6+3+3)*1

          13. 6+3+3+1

          14. (6*3)-(3+1)

          15. 13 + (6/3)

          16. 13 + (6-3)

          17. 33-16

          18. (1+3)*3 + 6

          19. (6 + 1/3)*3

          20. (3*6)+(3-1)

          21. (13-6)*3

          22. 13+3+6

          23. 36-13

          24. (6+1)*3 +3

          25. 16+(3*3)




          26-50 (except 41, 44, 46, 47)




          26. 33 - (6+1)

          27. 33 - (6*1)

          28. 61 - 33

          29. 31 - (6/3)

          30. (6+3+1)*3

          31. 13 + (6*3)

          32. 63-31

          33. 13*3 - 6

          34. 31+(6-3)

          35. 6*(3+3) -1

          36. (6+3)(3+1)

          37. 6
          (3+3) + 1

          38. 33+(6-1)

          39. 33+(6*1)

          40. 31+3+6

          41. round[36 + √(31)]

          42. (3+3+1)*6

          43. 16 + (3^3)

          44. round[(3-√3)^16]

          45. (3*13)+6

          46. round[3 * (13 + √6)]

          47. round[31√3] - 6

          48. (3*3 - 1) *6

          49. 36+13

          50. 63-13




          51-75 (except 56, 68, 69, 74)




          51. 16*3 + 3

          52. 61 - (3*3)

          53. (3*3*6)-1

          54. 1*3*3*6

          55. 61-(3+3)

          56. √(3136)     (!)

          57. (13+6)*3

          58. (3+1)^3 - 6

          59. 63 - (3+1)

          60. (63*1) - 3

          61. 61 + (3 - 3)

          62. 61 + (3/3)

          63. (6+1)*(3*3)

          64. 63 + (1^3)

          65. 63 + (3-1)

          66. 63 + (3*1)

          67. 36 + 31

          68. round[63 + √31]

          69. round[6 √133]

          70. 61+(3*3)

          71. (6^3)/3 - 1

          72. 36 * (3-1)

          73. (6^3)/3 + 1

          74. round[3 √613]

          75. 13*6 - 3




          76-100 (except: 77, 79, 83, 85*, 86, 89,90,91,92, 95, 97, 98, 100.)




          76. 63+13

          77. round[61 * (3-√3)]

          78. 13 * √(36)

          79. round[6*13 + √3]

          80. √(3^6) * 3 - 1

          81. 13*6 + 3

          82. √(3^6) * 3 + 1

          83. round[(3+31)*√6]

          84. 3^√(16) + 3

          85. -----

          86. round[√(6+√3) * 31]

          87. (31*3) - 6

          88. 61 + (3^3)

          89. round[63 * √(3-1)]

          90. round[13 * √(6/3)]

          91. round[16 * √33]

          92. round[3^√(3*6-1)]

          93. 31 * (6-3)

          94. 63+31

          95. round[3*31 + √6]

          96. (13+3)*6

          97. round[(√3)^6 * √13]

          98. round[36 * (1+√3)]

          99. (3*31)+6

          100. round[3*(31+√6)]




          If you're allowed to concatenate the results of operations (e.g. (3+1)|5 = 45 ) then a solution for 85 is :




          85. (3*3)|1 - 6




          Python:




          from itertools import permutations
          from math import sqrt, floor, ceil


          concat_literal_numbers_only = True

          ops = { "+" : lambda a,b: a+b,
          "-" : lambda a,b: a - b,
          "/" : lambda a,b : a/float(b),
          "*" : lambda a,b : a*b,
          "^" : lambda a,b : a**b,
          "C" : lambda a,b : float(str(a) + str(b)),
          "n" : lambda a : -a,
          "s" : lambda a : sqrt(a),
          #"f" : lambda a : floor(a)
          }

          arity = {"+" : 2,
          "-" : 2,
          "/" : 2,
          "*" : 2,
          "^" : 2,
          "C" : 2,
          "n" : 1,
          "s" : 1,
          "f" : 1,
          }


          # print ops["/"](1,3)


          # args: number of open args available
          # nums: available digits to be used
          # ops : tuple indicating commands used so far



          def evaluate(cmds) :
          """Consume the list of commands in prefix notation, producing a pair (ans, unconsumed_symbols)"""

          x = cmds.pop(0)
          if not ops.get(x) :
          return (x, cmds)
          else :
          args =
          for y in range(arity[x]) :
          try :
          (a, cmds) = evaluate(cmds)
          args += [a]
          except OverflowError :
          return (None, None)
          return (ops.get(x)(*args), cmds)

          def score(ops):
          ret = 0
          ret += ops.count("+")
          ret += 1.1*ops.count("-")
          ret += 2 * ops.count("*")
          ret += 3 * ops.count("/")
          ret += 3 * ops.count("n")
          ret += 4 * ops.count("^")
          ret += 4 * ops.count("s")
          ret += 4 * ops.count("f")
          ret += 4 * ops.count("w")

          # ret += 4 * ops.count("fs")
          # ret += 4 * ops.count("cs")
          return ret


          agenda = [{"args" : 1, "nums" : [1,3,3,6], "ops" : }]
          seen = {}


          only_search_for = None
          ret =

          def finish(ops) :
          global ret
          global seen

          ops_tmp = ops[:]
          try :
          n,_ = evaluate(ops_tmp)
          except :
          n = None

          if n is None or not (0 score(ops) :
          seen[n] = ops

          print ops,"t",n




          while agenda :
          x = agenda.pop(0)

          if not x["nums"] and not x["args"] : # finished: used up all numbers; no open spaces.
          finish(x["ops"])

          if len(x["nums"]) == x["args"] : # fill in numbers only
          for nums in set(permutations(x["nums"])) :
          finish(x["ops"] + list(nums))

          # print {"args" : 0,
          # "nums" : ,
          # "ops" : x["ops"] + list(nums)}

          elif len(x["nums"]) > x["args"] :
          # add new operators




          for op in ops.keys() :
          if arity[op] == 1 and x["ops"] and x["ops"][-1] == op :
          continue # limit repeated unary operations
          if arity[op] == 1 and x["ops"] and arity.get(x["ops"][-1]) == 1 :
          continue # limit repeated unary operations


          if (concat_literal_numbers_only and x["ops"] and (x["ops"][-1] == "C" or (len(x["ops"])>1 and x["ops"][-2] == "C")) and op != "C") :
          continue


          new_x = {"args" : x["args"] + arity[op] - 1,
          "nums" : x["nums"],
          "ops" : x["ops"] + [op]}
          agenda = [new_x] + agenda

          if x["args"] == 1 :
          continue

          for n in set(x["nums"]) :
          new_nums = x["nums"][:]
          new_nums.remove(n)
          new_x = {"args" : x["args"] - 1,
          "nums" : new_nums,
          "ops" : x["ops"] + [n]}

          agenda = [new_x] + agenda

          # SHOW HOW TO MAKE ALL OF THE NUMBERS
          miss =
          for i in range(0+1,100+1) :
          if not seen.get(i) :
          miss += [i]
          print i, "t", seen.get(i, "---")

          # SHOW WHICH NUMBERS WERE MISSED
          print "missed: ", miss

          # IF YOU'RE LOOKING FOR ALL POSSIBLE WAYS TO MAKE SOMETHING, SHOW THEM HERE.
          if only_search_for is not None :
          ret = sorted(ret, key=score)
          for x in ret:
          print x





          share|improve this answer











          $endgroup$



          Here's a solution for most of them. The remaining 21 are impossible.




          • Most of them only use simple arithmetic.

          • Some of them use exponentiation.

          • Some use square roots.


          Edit: Using only the binary operators (including digit concatenation), the number of possible combinations is pretty small (arrange three binary operators, then fill in the four numbers in some order) and I double-checked all of them with a computer. Most numbers are solvable with basic arithmetic, some require exponentiation, some require negation or square roots, and the rest are apparently impossible to solve without some extra operation such as rounding.



          Accordingly, it is provably impossible to construct the following twenty-one numbers:




          41, 44, 46, 47, 56, 68, 69, 74, 77, 79, 83, 85, 86, 89, 90, 91, 92, 95, 97, 98, 100.




          The rest can be constructed as follows:



          1-25 (complete)




          1. (6+1)-(3+3)

          2. (3*3)-(6+1)

          3. (3*3)-(6*1)

          4. 13-(6+3)

          5. 36-31

          6. 3*(3+1)-6

          7. (6+1)+(3-3)

          8. (6-3)3-1

          9. 6
          (3-1)-3

          10. 3+13-6

          11. 6+3+3-1

          12. (6+3+3)*1

          13. 6+3+3+1

          14. (6*3)-(3+1)

          15. 13 + (6/3)

          16. 13 + (6-3)

          17. 33-16

          18. (1+3)*3 + 6

          19. (6 + 1/3)*3

          20. (3*6)+(3-1)

          21. (13-6)*3

          22. 13+3+6

          23. 36-13

          24. (6+1)*3 +3

          25. 16+(3*3)




          26-50 (except 41, 44, 46, 47)




          26. 33 - (6+1)

          27. 33 - (6*1)

          28. 61 - 33

          29. 31 - (6/3)

          30. (6+3+1)*3

          31. 13 + (6*3)

          32. 63-31

          33. 13*3 - 6

          34. 31+(6-3)

          35. 6*(3+3) -1

          36. (6+3)(3+1)

          37. 6
          (3+3) + 1

          38. 33+(6-1)

          39. 33+(6*1)

          40. 31+3+6

          41. round[36 + √(31)]

          42. (3+3+1)*6

          43. 16 + (3^3)

          44. round[(3-√3)^16]

          45. (3*13)+6

          46. round[3 * (13 + √6)]

          47. round[31√3] - 6

          48. (3*3 - 1) *6

          49. 36+13

          50. 63-13




          51-75 (except 56, 68, 69, 74)




          51. 16*3 + 3

          52. 61 - (3*3)

          53. (3*3*6)-1

          54. 1*3*3*6

          55. 61-(3+3)

          56. √(3136)     (!)

          57. (13+6)*3

          58. (3+1)^3 - 6

          59. 63 - (3+1)

          60. (63*1) - 3

          61. 61 + (3 - 3)

          62. 61 + (3/3)

          63. (6+1)*(3*3)

          64. 63 + (1^3)

          65. 63 + (3-1)

          66. 63 + (3*1)

          67. 36 + 31

          68. round[63 + √31]

          69. round[6 √133]

          70. 61+(3*3)

          71. (6^3)/3 - 1

          72. 36 * (3-1)

          73. (6^3)/3 + 1

          74. round[3 √613]

          75. 13*6 - 3




          76-100 (except: 77, 79, 83, 85*, 86, 89,90,91,92, 95, 97, 98, 100.)




          76. 63+13

          77. round[61 * (3-√3)]

          78. 13 * √(36)

          79. round[6*13 + √3]

          80. √(3^6) * 3 - 1

          81. 13*6 + 3

          82. √(3^6) * 3 + 1

          83. round[(3+31)*√6]

          84. 3^√(16) + 3

          85. -----

          86. round[√(6+√3) * 31]

          87. (31*3) - 6

          88. 61 + (3^3)

          89. round[63 * √(3-1)]

          90. round[13 * √(6/3)]

          91. round[16 * √33]

          92. round[3^√(3*6-1)]

          93. 31 * (6-3)

          94. 63+31

          95. round[3*31 + √6]

          96. (13+3)*6

          97. round[(√3)^6 * √13]

          98. round[36 * (1+√3)]

          99. (3*31)+6

          100. round[3*(31+√6)]




          If you're allowed to concatenate the results of operations (e.g. (3+1)|5 = 45 ) then a solution for 85 is :




          85. (3*3)|1 - 6




          Python:




          from itertools import permutations
          from math import sqrt, floor, ceil


          concat_literal_numbers_only = True

          ops = { "+" : lambda a,b: a+b,
          "-" : lambda a,b: a - b,
          "/" : lambda a,b : a/float(b),
          "*" : lambda a,b : a*b,
          "^" : lambda a,b : a**b,
          "C" : lambda a,b : float(str(a) + str(b)),
          "n" : lambda a : -a,
          "s" : lambda a : sqrt(a),
          #"f" : lambda a : floor(a)
          }

          arity = {"+" : 2,
          "-" : 2,
          "/" : 2,
          "*" : 2,
          "^" : 2,
          "C" : 2,
          "n" : 1,
          "s" : 1,
          "f" : 1,
          }


          # print ops["/"](1,3)


          # args: number of open args available
          # nums: available digits to be used
          # ops : tuple indicating commands used so far



          def evaluate(cmds) :
          """Consume the list of commands in prefix notation, producing a pair (ans, unconsumed_symbols)"""

          x = cmds.pop(0)
          if not ops.get(x) :
          return (x, cmds)
          else :
          args =
          for y in range(arity[x]) :
          try :
          (a, cmds) = evaluate(cmds)
          args += [a]
          except OverflowError :
          return (None, None)
          return (ops.get(x)(*args), cmds)

          def score(ops):
          ret = 0
          ret += ops.count("+")
          ret += 1.1*ops.count("-")
          ret += 2 * ops.count("*")
          ret += 3 * ops.count("/")
          ret += 3 * ops.count("n")
          ret += 4 * ops.count("^")
          ret += 4 * ops.count("s")
          ret += 4 * ops.count("f")
          ret += 4 * ops.count("w")

          # ret += 4 * ops.count("fs")
          # ret += 4 * ops.count("cs")
          return ret


          agenda = [{"args" : 1, "nums" : [1,3,3,6], "ops" : }]
          seen = {}


          only_search_for = None
          ret =

          def finish(ops) :
          global ret
          global seen

          ops_tmp = ops[:]
          try :
          n,_ = evaluate(ops_tmp)
          except :
          n = None

          if n is None or not (0 score(ops) :
          seen[n] = ops

          print ops,"t",n




          while agenda :
          x = agenda.pop(0)

          if not x["nums"] and not x["args"] : # finished: used up all numbers; no open spaces.
          finish(x["ops"])

          if len(x["nums"]) == x["args"] : # fill in numbers only
          for nums in set(permutations(x["nums"])) :
          finish(x["ops"] + list(nums))

          # print {"args" : 0,
          # "nums" : ,
          # "ops" : x["ops"] + list(nums)}

          elif len(x["nums"]) > x["args"] :
          # add new operators




          for op in ops.keys() :
          if arity[op] == 1 and x["ops"] and x["ops"][-1] == op :
          continue # limit repeated unary operations
          if arity[op] == 1 and x["ops"] and arity.get(x["ops"][-1]) == 1 :
          continue # limit repeated unary operations


          if (concat_literal_numbers_only and x["ops"] and (x["ops"][-1] == "C" or (len(x["ops"])>1 and x["ops"][-2] == "C")) and op != "C") :
          continue


          new_x = {"args" : x["args"] + arity[op] - 1,
          "nums" : x["nums"],
          "ops" : x["ops"] + [op]}
          agenda = [new_x] + agenda

          if x["args"] == 1 :
          continue

          for n in set(x["nums"]) :
          new_nums = x["nums"][:]
          new_nums.remove(n)
          new_x = {"args" : x["args"] - 1,
          "nums" : new_nums,
          "ops" : x["ops"] + [n]}

          agenda = [new_x] + agenda

          # SHOW HOW TO MAKE ALL OF THE NUMBERS
          miss =
          for i in range(0+1,100+1) :
          if not seen.get(i) :
          miss += [i]
          print i, "t", seen.get(i, "---")

          # SHOW WHICH NUMBERS WERE MISSED
          print "missed: ", miss

          # IF YOU'RE LOOKING FOR ALL POSSIBLE WAYS TO MAKE SOMETHING, SHOW THEM HERE.
          if only_search_for is not None :
          ret = sorted(ret, key=score)
          for x in ret:
          print x






          share|improve this answer














          share|improve this answer



          share|improve this answer








          edited Feb 1 at 2:09

























          answered Jan 28 at 7:27









          user326210user326210

          1712




          1712












          • $begingroup$
            Although it's unlikely that you'd find the missing solutions there, looks like the code ignores Nth roots entirely.
            $endgroup$
            – Bass
            Jan 29 at 9:47










          • $begingroup$
            @Bass . Thanks, yes ---- when posting the code I took out some operators that didn't add to the number of solutions.
            $endgroup$
            – user326210
            Jan 30 at 1:00


















          • $begingroup$
            Although it's unlikely that you'd find the missing solutions there, looks like the code ignores Nth roots entirely.
            $endgroup$
            – Bass
            Jan 29 at 9:47










          • $begingroup$
            @Bass . Thanks, yes ---- when posting the code I took out some operators that didn't add to the number of solutions.
            $endgroup$
            – user326210
            Jan 30 at 1:00
















          $begingroup$
          Although it's unlikely that you'd find the missing solutions there, looks like the code ignores Nth roots entirely.
          $endgroup$
          – Bass
          Jan 29 at 9:47




          $begingroup$
          Although it's unlikely that you'd find the missing solutions there, looks like the code ignores Nth roots entirely.
          $endgroup$
          – Bass
          Jan 29 at 9:47












          $begingroup$
          @Bass . Thanks, yes ---- when posting the code I took out some operators that didn't add to the number of solutions.
          $endgroup$
          – user326210
          Jan 30 at 1:00




          $begingroup$
          @Bass . Thanks, yes ---- when posting the code I took out some operators that didn't add to the number of solutions.
          $endgroup$
          – user326210
          Jan 30 at 1:00











          3












          $begingroup$

          We can generate any integer using only $1$, $3$, $3$ and $6$ with the introduction of one special function.




          The function in question is the Logarithm to an arbitrary base $b$ , or $log _{b} (x)$.



          To begin, let's discuss square root stacking.
          $sqrt{sqrt{a}}$ is equivalent to $sqrt[4]{a}$, and $sqrt{sqrt{sqrt{a}}}$ is equivalent to $sqrt[8]{a}$, which can be rewritten as $a^frac{1}{8}$. This pattern continues indefinitely; $a$ with $n$ square roots stacked to it is equal to $a^frac{1}{2^n}$.


          The laws of logarithms state that $log _{b} (x^a) = a cdot log _{b} (x)$. If we take the logarithm to base $b$ or our previous square root stack, we get $log _{b} (a^frac{1}{2^n})$, or $frac{1}{2^n} cdot log _{b}a$. Setting $a$ and $b$ as 3 means that $log _{3} (sqrt{sqrt[...]{3}})$, with $n$ square roots, is equal to $frac{1}{2^n} cdot log _{3}3$, or $2^{-n}$. $(frac{1}{a^x} = a^{-x})$

          $sqrt{sqrt{16}} = 2$, and $log _{b}(b^a) = a$. As such,

          $0 = -log _{sqrt{sqrt{16}}}(log_{3}(3))$
          $1 = -log _{sqrt{sqrt{16}}}(log_{3}(sqrt{3}))$
          $2 = -log _{sqrt{sqrt{16}}}(log_{3}(sqrt{sqrt{3}}))$
          $3 = -log _{sqrt{sqrt{16}}}(log_{3}(sqrt{sqrt{sqrt{3}}}))$
          $4 = -log _{sqrt{sqrt{16}}}(log_{3}(sqrt{sqrt{sqrt{sqrt{3}}}}))$

          And so on, such that the amount of square roots is equal to your desired number.


          This works for all integers; for negative numbers simply remove the $-$ at the start.




          This was inspired by Numberphile's video on the four 4s.






          share|improve this answer









          $endgroup$


















            3












            $begingroup$

            We can generate any integer using only $1$, $3$, $3$ and $6$ with the introduction of one special function.




            The function in question is the Logarithm to an arbitrary base $b$ , or $log _{b} (x)$.



            To begin, let's discuss square root stacking.
            $sqrt{sqrt{a}}$ is equivalent to $sqrt[4]{a}$, and $sqrt{sqrt{sqrt{a}}}$ is equivalent to $sqrt[8]{a}$, which can be rewritten as $a^frac{1}{8}$. This pattern continues indefinitely; $a$ with $n$ square roots stacked to it is equal to $a^frac{1}{2^n}$.


            The laws of logarithms state that $log _{b} (x^a) = a cdot log _{b} (x)$. If we take the logarithm to base $b$ or our previous square root stack, we get $log _{b} (a^frac{1}{2^n})$, or $frac{1}{2^n} cdot log _{b}a$. Setting $a$ and $b$ as 3 means that $log _{3} (sqrt{sqrt[...]{3}})$, with $n$ square roots, is equal to $frac{1}{2^n} cdot log _{3}3$, or $2^{-n}$. $(frac{1}{a^x} = a^{-x})$

            $sqrt{sqrt{16}} = 2$, and $log _{b}(b^a) = a$. As such,

            $0 = -log _{sqrt{sqrt{16}}}(log_{3}(3))$
            $1 = -log _{sqrt{sqrt{16}}}(log_{3}(sqrt{3}))$
            $2 = -log _{sqrt{sqrt{16}}}(log_{3}(sqrt{sqrt{3}}))$
            $3 = -log _{sqrt{sqrt{16}}}(log_{3}(sqrt{sqrt{sqrt{3}}}))$
            $4 = -log _{sqrt{sqrt{16}}}(log_{3}(sqrt{sqrt{sqrt{sqrt{3}}}}))$

            And so on, such that the amount of square roots is equal to your desired number.


            This works for all integers; for negative numbers simply remove the $-$ at the start.




            This was inspired by Numberphile's video on the four 4s.






            share|improve this answer









            $endgroup$
















              3












              3








              3





              $begingroup$

              We can generate any integer using only $1$, $3$, $3$ and $6$ with the introduction of one special function.




              The function in question is the Logarithm to an arbitrary base $b$ , or $log _{b} (x)$.



              To begin, let's discuss square root stacking.
              $sqrt{sqrt{a}}$ is equivalent to $sqrt[4]{a}$, and $sqrt{sqrt{sqrt{a}}}$ is equivalent to $sqrt[8]{a}$, which can be rewritten as $a^frac{1}{8}$. This pattern continues indefinitely; $a$ with $n$ square roots stacked to it is equal to $a^frac{1}{2^n}$.


              The laws of logarithms state that $log _{b} (x^a) = a cdot log _{b} (x)$. If we take the logarithm to base $b$ or our previous square root stack, we get $log _{b} (a^frac{1}{2^n})$, or $frac{1}{2^n} cdot log _{b}a$. Setting $a$ and $b$ as 3 means that $log _{3} (sqrt{sqrt[...]{3}})$, with $n$ square roots, is equal to $frac{1}{2^n} cdot log _{3}3$, or $2^{-n}$. $(frac{1}{a^x} = a^{-x})$

              $sqrt{sqrt{16}} = 2$, and $log _{b}(b^a) = a$. As such,

              $0 = -log _{sqrt{sqrt{16}}}(log_{3}(3))$
              $1 = -log _{sqrt{sqrt{16}}}(log_{3}(sqrt{3}))$
              $2 = -log _{sqrt{sqrt{16}}}(log_{3}(sqrt{sqrt{3}}))$
              $3 = -log _{sqrt{sqrt{16}}}(log_{3}(sqrt{sqrt{sqrt{3}}}))$
              $4 = -log _{sqrt{sqrt{16}}}(log_{3}(sqrt{sqrt{sqrt{sqrt{3}}}}))$

              And so on, such that the amount of square roots is equal to your desired number.


              This works for all integers; for negative numbers simply remove the $-$ at the start.




              This was inspired by Numberphile's video on the four 4s.






              share|improve this answer









              $endgroup$



              We can generate any integer using only $1$, $3$, $3$ and $6$ with the introduction of one special function.




              The function in question is the Logarithm to an arbitrary base $b$ , or $log _{b} (x)$.



              To begin, let's discuss square root stacking.
              $sqrt{sqrt{a}}$ is equivalent to $sqrt[4]{a}$, and $sqrt{sqrt{sqrt{a}}}$ is equivalent to $sqrt[8]{a}$, which can be rewritten as $a^frac{1}{8}$. This pattern continues indefinitely; $a$ with $n$ square roots stacked to it is equal to $a^frac{1}{2^n}$.


              The laws of logarithms state that $log _{b} (x^a) = a cdot log _{b} (x)$. If we take the logarithm to base $b$ or our previous square root stack, we get $log _{b} (a^frac{1}{2^n})$, or $frac{1}{2^n} cdot log _{b}a$. Setting $a$ and $b$ as 3 means that $log _{3} (sqrt{sqrt[...]{3}})$, with $n$ square roots, is equal to $frac{1}{2^n} cdot log _{3}3$, or $2^{-n}$. $(frac{1}{a^x} = a^{-x})$

              $sqrt{sqrt{16}} = 2$, and $log _{b}(b^a) = a$. As such,

              $0 = -log _{sqrt{sqrt{16}}}(log_{3}(3))$
              $1 = -log _{sqrt{sqrt{16}}}(log_{3}(sqrt{3}))$
              $2 = -log _{sqrt{sqrt{16}}}(log_{3}(sqrt{sqrt{3}}))$
              $3 = -log _{sqrt{sqrt{16}}}(log_{3}(sqrt{sqrt{sqrt{3}}}))$
              $4 = -log _{sqrt{sqrt{16}}}(log_{3}(sqrt{sqrt{sqrt{sqrt{3}}}}))$

              And so on, such that the amount of square roots is equal to your desired number.


              This works for all integers; for negative numbers simply remove the $-$ at the start.




              This was inspired by Numberphile's video on the four 4s.







              share|improve this answer












              share|improve this answer



              share|improve this answer










              answered Jan 28 at 14:27









              user56759user56759

              311




              311























                  3












                  $begingroup$

                  Here are some:




                  1: $3 + 3 - 6 + 1$

                  2: $3 * 3 - (6 + 1)$

                  3: $3 * 3 * 1 - 6$

                  4: $3 * 3 - 6 + 1$

                  5: $(3 * 6) / 3 - 1$

                  6: $(3 * 6) / 3 * 1$

                  7: $(3 * 6) / 3 + 1$

                  8: $3 * 3 - 1 ^ 6$

                  9: $(3 * 6) / (3 - 1)$

                  10: $3 * 3 + 1 ^ 6$

                  11: $36 / 3 - 1$

                  12: $36 / 3 * 1$

                  13: $36 / 3 + 1$

                  14: $3 * 6 - (3 + 1)$

                  15: $3 * 6 - (3 * 1)$

                  16: $3 * 6 - (3 - 1)$

                  17: $3 * 6 - 1 ^ 3$

                  18: $3 * 6 * 1 ^ 3$

                  19: $3 * 6 + 1 ^ 3$

                  20: $3 * 6 + 3 - 1$

                  22: (omega kyrpton did some) $3 * 6 + 3 + 1$




                  I will do more later.






                  share|improve this answer











                  $endgroup$


















                    3












                    $begingroup$

                    Here are some:




                    1: $3 + 3 - 6 + 1$

                    2: $3 * 3 - (6 + 1)$

                    3: $3 * 3 * 1 - 6$

                    4: $3 * 3 - 6 + 1$

                    5: $(3 * 6) / 3 - 1$

                    6: $(3 * 6) / 3 * 1$

                    7: $(3 * 6) / 3 + 1$

                    8: $3 * 3 - 1 ^ 6$

                    9: $(3 * 6) / (3 - 1)$

                    10: $3 * 3 + 1 ^ 6$

                    11: $36 / 3 - 1$

                    12: $36 / 3 * 1$

                    13: $36 / 3 + 1$

                    14: $3 * 6 - (3 + 1)$

                    15: $3 * 6 - (3 * 1)$

                    16: $3 * 6 - (3 - 1)$

                    17: $3 * 6 - 1 ^ 3$

                    18: $3 * 6 * 1 ^ 3$

                    19: $3 * 6 + 1 ^ 3$

                    20: $3 * 6 + 3 - 1$

                    22: (omega kyrpton did some) $3 * 6 + 3 + 1$




                    I will do more later.






                    share|improve this answer











                    $endgroup$
















                      3












                      3








                      3





                      $begingroup$

                      Here are some:




                      1: $3 + 3 - 6 + 1$

                      2: $3 * 3 - (6 + 1)$

                      3: $3 * 3 * 1 - 6$

                      4: $3 * 3 - 6 + 1$

                      5: $(3 * 6) / 3 - 1$

                      6: $(3 * 6) / 3 * 1$

                      7: $(3 * 6) / 3 + 1$

                      8: $3 * 3 - 1 ^ 6$

                      9: $(3 * 6) / (3 - 1)$

                      10: $3 * 3 + 1 ^ 6$

                      11: $36 / 3 - 1$

                      12: $36 / 3 * 1$

                      13: $36 / 3 + 1$

                      14: $3 * 6 - (3 + 1)$

                      15: $3 * 6 - (3 * 1)$

                      16: $3 * 6 - (3 - 1)$

                      17: $3 * 6 - 1 ^ 3$

                      18: $3 * 6 * 1 ^ 3$

                      19: $3 * 6 + 1 ^ 3$

                      20: $3 * 6 + 3 - 1$

                      22: (omega kyrpton did some) $3 * 6 + 3 + 1$




                      I will do more later.






                      share|improve this answer











                      $endgroup$



                      Here are some:




                      1: $3 + 3 - 6 + 1$

                      2: $3 * 3 - (6 + 1)$

                      3: $3 * 3 * 1 - 6$

                      4: $3 * 3 - 6 + 1$

                      5: $(3 * 6) / 3 - 1$

                      6: $(3 * 6) / 3 * 1$

                      7: $(3 * 6) / 3 + 1$

                      8: $3 * 3 - 1 ^ 6$

                      9: $(3 * 6) / (3 - 1)$

                      10: $3 * 3 + 1 ^ 6$

                      11: $36 / 3 - 1$

                      12: $36 / 3 * 1$

                      13: $36 / 3 + 1$

                      14: $3 * 6 - (3 + 1)$

                      15: $3 * 6 - (3 * 1)$

                      16: $3 * 6 - (3 - 1)$

                      17: $3 * 6 - 1 ^ 3$

                      18: $3 * 6 * 1 ^ 3$

                      19: $3 * 6 + 1 ^ 3$

                      20: $3 * 6 + 3 - 1$

                      22: (omega kyrpton did some) $3 * 6 + 3 + 1$




                      I will do more later.







                      share|improve this answer














                      share|improve this answer



                      share|improve this answer








                      edited Jan 28 at 15:54

























                      answered Jan 27 at 22:33









                      Yout RiedYout Ried

                      894119




                      894119























                          2












                          $begingroup$

                          Adding some more...



                          1-20: (Credits to @YoutRied)




                          1: $3 + 3 - 6 + 1$

                          2: $3 * 3 - (6 + 1)$

                          3: $3 * 3 * 1 - 6$

                          4: $3 * 3 - 6 + 1$

                          5: $(3 * 6) / 3 - 1$

                          6: $(3 * 6) / 3 * 1$

                          7: $(3 * 6) / 3 + 1$

                          8: $3 * 3 - 1 ^ 6$

                          9: $(3 * 6) / (3 - 1)$

                          10: $3 * 3 + 1 ^ 6$

                          11: $36 / 3 - 1$

                          12: $36 / 3 * 1$

                          13: $36 / 3 + 1$

                          14: $3 * 6 - (3 + 1)$

                          15: $3 * 6 - (3 * 1)$

                          16: $3 * 6 - (3 - 1)$

                          17: $3 * 6 - 1 ^ 3$

                          18: $3 * 6 * 1 ^ 3$

                          19: $3 * 6 + 1 ^ 3$

                          20: $3 * 6 + 3 - 1$




                          21-29




                          21: $3 * 6 + 3 * 1$

                          22: $( 1 + 3 ) ! - ( 6 / 3 )$

                          23: $( 1 + 3 ) ! - ( 6 - 3 )$

                          24: $( 6 - 3 / 3 - 1 ) !$

                          25: $1 * 3 ^ 3 - floor(sqrt{6})$

                          26: $( 6 - 3 ) ^ 3 - 1$

                          27: $( 6 - 3 ) ^ 3 * 1$

                          28: $( 6 - 3 ) ^ 3 + 1$

                          29: $31 - 6 / 3$




                          41-50: (Credits to @Bass for 42, 45, 49)




                          41: $ (-1+3!)+36 $

                          42: $ (1+3+3)times 6$

                          43: $ 31 + 6 * ceil(sqrt{3})$

                          44: $floor( 1 * 3 * sqrt{6 ^ 3}) $

                          45: $ 13times3+6$

                          46: $ ceil(sqrt{6 ^ 3} + 31)$

                          47: $ floor(sqrt{sqrt{sqrt{sqrt{sqrt{31!}}}}})+36$

                          48: $6 * ( 3 * 3 - 1 )$

                          49: $13+36$

                          50: $ (6+1)^2 + 3 - 3$




                          51-60:




                          51: $( 3 * 6 - 1 ) * 3$

                          52: $( 3 + 3 + 1 ) * ceil(sqrt{6})$

                          53: $-1+( 3 * 3 * 6 )$

                          54: $ 1*3 * 3 * 6 $

                          55: $1+3*3*6$

                          56: $61-3!+floor(sqrt{3})$

                          57: $1*63-3!$

                          58: $1+63-3!$

                          59: $floor(sqrt{sqrt{sqrt{sqrt{sqrt{6^{(3-1)}}}}}}*3)$

                          60: $(1+3*3)*6$




                          61-70:




                          61: $63-3+1$

                          62: $63+1-ceil(sqrt{3})$

                          63: $63-floor(sqrt{3})+1$

                          64: $63+ceil(sqrt{3})-1$

                          65: $63+3-1$

                          66: $63+3*1$

                          67: $63+3+1$

                          68: $61+3!+floor(sqrt{3})$

                          69: $61+3!+ceil(sqrt{3})$

                          70: $61+3*3$




                          71-80




                          71: $(3+1)!*3-floor(sqrt{sqrt{6}})$

                          72: $(3+1)*3*6$

                          73: $(3+1)!*3+floor(sqrt{sqrt{6}})$

                          74: $(3+1)!*3+floor(sqrt{6})$

                          75: $(3+1)!*3+ceil(sqrt{6})$

                          76: $ceil(sqrt{sqrt{sqrt{ceil(sqrt{sqrt{sqrt{sqrt{sqrt{ceil(sqrt{3!!})!}}}}})}}})*(3*6+1)$

                          76:(much simpler) $13*6-ceil(sqrt{3})$

                          77: $13*6-floor(sqrt{3})$

                          78: $13*floor(sqrt{3})*6$

                          79: $13*6+floor(sqrt{3})$

                          80: $13*6+ceil(sqrt{3})$




                          81-90:




                          81: $(6+3)^{(3-1)}$

                          82: $(6+3)^{ceil(sqrt{3})}+1$







                          share|improve this answer











                          $endgroup$













                          • $begingroup$
                            Who said you could use factorials?
                            $endgroup$
                            – Yout Ried
                            Jan 28 at 0:20










                          • $begingroup$
                            What are number 23 (plus you probably can't use factorials and 24? I don't get them.
                            $endgroup$
                            – Yout Ried
                            Jan 28 at 1:08












                          • $begingroup$
                            Oops forgot a ")" and maybe you're missing a factorial for number 24
                            $endgroup$
                            – Yout Ried
                            Jan 28 at 1:16
















                          2












                          $begingroup$

                          Adding some more...



                          1-20: (Credits to @YoutRied)




                          1: $3 + 3 - 6 + 1$

                          2: $3 * 3 - (6 + 1)$

                          3: $3 * 3 * 1 - 6$

                          4: $3 * 3 - 6 + 1$

                          5: $(3 * 6) / 3 - 1$

                          6: $(3 * 6) / 3 * 1$

                          7: $(3 * 6) / 3 + 1$

                          8: $3 * 3 - 1 ^ 6$

                          9: $(3 * 6) / (3 - 1)$

                          10: $3 * 3 + 1 ^ 6$

                          11: $36 / 3 - 1$

                          12: $36 / 3 * 1$

                          13: $36 / 3 + 1$

                          14: $3 * 6 - (3 + 1)$

                          15: $3 * 6 - (3 * 1)$

                          16: $3 * 6 - (3 - 1)$

                          17: $3 * 6 - 1 ^ 3$

                          18: $3 * 6 * 1 ^ 3$

                          19: $3 * 6 + 1 ^ 3$

                          20: $3 * 6 + 3 - 1$




                          21-29




                          21: $3 * 6 + 3 * 1$

                          22: $( 1 + 3 ) ! - ( 6 / 3 )$

                          23: $( 1 + 3 ) ! - ( 6 - 3 )$

                          24: $( 6 - 3 / 3 - 1 ) !$

                          25: $1 * 3 ^ 3 - floor(sqrt{6})$

                          26: $( 6 - 3 ) ^ 3 - 1$

                          27: $( 6 - 3 ) ^ 3 * 1$

                          28: $( 6 - 3 ) ^ 3 + 1$

                          29: $31 - 6 / 3$




                          41-50: (Credits to @Bass for 42, 45, 49)




                          41: $ (-1+3!)+36 $

                          42: $ (1+3+3)times 6$

                          43: $ 31 + 6 * ceil(sqrt{3})$

                          44: $floor( 1 * 3 * sqrt{6 ^ 3}) $

                          45: $ 13times3+6$

                          46: $ ceil(sqrt{6 ^ 3} + 31)$

                          47: $ floor(sqrt{sqrt{sqrt{sqrt{sqrt{31!}}}}})+36$

                          48: $6 * ( 3 * 3 - 1 )$

                          49: $13+36$

                          50: $ (6+1)^2 + 3 - 3$




                          51-60:




                          51: $( 3 * 6 - 1 ) * 3$

                          52: $( 3 + 3 + 1 ) * ceil(sqrt{6})$

                          53: $-1+( 3 * 3 * 6 )$

                          54: $ 1*3 * 3 * 6 $

                          55: $1+3*3*6$

                          56: $61-3!+floor(sqrt{3})$

                          57: $1*63-3!$

                          58: $1+63-3!$

                          59: $floor(sqrt{sqrt{sqrt{sqrt{sqrt{6^{(3-1)}}}}}}*3)$

                          60: $(1+3*3)*6$




                          61-70:




                          61: $63-3+1$

                          62: $63+1-ceil(sqrt{3})$

                          63: $63-floor(sqrt{3})+1$

                          64: $63+ceil(sqrt{3})-1$

                          65: $63+3-1$

                          66: $63+3*1$

                          67: $63+3+1$

                          68: $61+3!+floor(sqrt{3})$

                          69: $61+3!+ceil(sqrt{3})$

                          70: $61+3*3$




                          71-80




                          71: $(3+1)!*3-floor(sqrt{sqrt{6}})$

                          72: $(3+1)*3*6$

                          73: $(3+1)!*3+floor(sqrt{sqrt{6}})$

                          74: $(3+1)!*3+floor(sqrt{6})$

                          75: $(3+1)!*3+ceil(sqrt{6})$

                          76: $ceil(sqrt{sqrt{sqrt{ceil(sqrt{sqrt{sqrt{sqrt{sqrt{ceil(sqrt{3!!})!}}}}})}}})*(3*6+1)$

                          76:(much simpler) $13*6-ceil(sqrt{3})$

                          77: $13*6-floor(sqrt{3})$

                          78: $13*floor(sqrt{3})*6$

                          79: $13*6+floor(sqrt{3})$

                          80: $13*6+ceil(sqrt{3})$




                          81-90:




                          81: $(6+3)^{(3-1)}$

                          82: $(6+3)^{ceil(sqrt{3})}+1$







                          share|improve this answer











                          $endgroup$













                          • $begingroup$
                            Who said you could use factorials?
                            $endgroup$
                            – Yout Ried
                            Jan 28 at 0:20










                          • $begingroup$
                            What are number 23 (plus you probably can't use factorials and 24? I don't get them.
                            $endgroup$
                            – Yout Ried
                            Jan 28 at 1:08












                          • $begingroup$
                            Oops forgot a ")" and maybe you're missing a factorial for number 24
                            $endgroup$
                            – Yout Ried
                            Jan 28 at 1:16














                          2












                          2








                          2





                          $begingroup$

                          Adding some more...



                          1-20: (Credits to @YoutRied)




                          1: $3 + 3 - 6 + 1$

                          2: $3 * 3 - (6 + 1)$

                          3: $3 * 3 * 1 - 6$

                          4: $3 * 3 - 6 + 1$

                          5: $(3 * 6) / 3 - 1$

                          6: $(3 * 6) / 3 * 1$

                          7: $(3 * 6) / 3 + 1$

                          8: $3 * 3 - 1 ^ 6$

                          9: $(3 * 6) / (3 - 1)$

                          10: $3 * 3 + 1 ^ 6$

                          11: $36 / 3 - 1$

                          12: $36 / 3 * 1$

                          13: $36 / 3 + 1$

                          14: $3 * 6 - (3 + 1)$

                          15: $3 * 6 - (3 * 1)$

                          16: $3 * 6 - (3 - 1)$

                          17: $3 * 6 - 1 ^ 3$

                          18: $3 * 6 * 1 ^ 3$

                          19: $3 * 6 + 1 ^ 3$

                          20: $3 * 6 + 3 - 1$




                          21-29




                          21: $3 * 6 + 3 * 1$

                          22: $( 1 + 3 ) ! - ( 6 / 3 )$

                          23: $( 1 + 3 ) ! - ( 6 - 3 )$

                          24: $( 6 - 3 / 3 - 1 ) !$

                          25: $1 * 3 ^ 3 - floor(sqrt{6})$

                          26: $( 6 - 3 ) ^ 3 - 1$

                          27: $( 6 - 3 ) ^ 3 * 1$

                          28: $( 6 - 3 ) ^ 3 + 1$

                          29: $31 - 6 / 3$




                          41-50: (Credits to @Bass for 42, 45, 49)




                          41: $ (-1+3!)+36 $

                          42: $ (1+3+3)times 6$

                          43: $ 31 + 6 * ceil(sqrt{3})$

                          44: $floor( 1 * 3 * sqrt{6 ^ 3}) $

                          45: $ 13times3+6$

                          46: $ ceil(sqrt{6 ^ 3} + 31)$

                          47: $ floor(sqrt{sqrt{sqrt{sqrt{sqrt{31!}}}}})+36$

                          48: $6 * ( 3 * 3 - 1 )$

                          49: $13+36$

                          50: $ (6+1)^2 + 3 - 3$




                          51-60:




                          51: $( 3 * 6 - 1 ) * 3$

                          52: $( 3 + 3 + 1 ) * ceil(sqrt{6})$

                          53: $-1+( 3 * 3 * 6 )$

                          54: $ 1*3 * 3 * 6 $

                          55: $1+3*3*6$

                          56: $61-3!+floor(sqrt{3})$

                          57: $1*63-3!$

                          58: $1+63-3!$

                          59: $floor(sqrt{sqrt{sqrt{sqrt{sqrt{6^{(3-1)}}}}}}*3)$

                          60: $(1+3*3)*6$




                          61-70:




                          61: $63-3+1$

                          62: $63+1-ceil(sqrt{3})$

                          63: $63-floor(sqrt{3})+1$

                          64: $63+ceil(sqrt{3})-1$

                          65: $63+3-1$

                          66: $63+3*1$

                          67: $63+3+1$

                          68: $61+3!+floor(sqrt{3})$

                          69: $61+3!+ceil(sqrt{3})$

                          70: $61+3*3$




                          71-80




                          71: $(3+1)!*3-floor(sqrt{sqrt{6}})$

                          72: $(3+1)*3*6$

                          73: $(3+1)!*3+floor(sqrt{sqrt{6}})$

                          74: $(3+1)!*3+floor(sqrt{6})$

                          75: $(3+1)!*3+ceil(sqrt{6})$

                          76: $ceil(sqrt{sqrt{sqrt{ceil(sqrt{sqrt{sqrt{sqrt{sqrt{ceil(sqrt{3!!})!}}}}})}}})*(3*6+1)$

                          76:(much simpler) $13*6-ceil(sqrt{3})$

                          77: $13*6-floor(sqrt{3})$

                          78: $13*floor(sqrt{3})*6$

                          79: $13*6+floor(sqrt{3})$

                          80: $13*6+ceil(sqrt{3})$




                          81-90:




                          81: $(6+3)^{(3-1)}$

                          82: $(6+3)^{ceil(sqrt{3})}+1$







                          share|improve this answer











                          $endgroup$



                          Adding some more...



                          1-20: (Credits to @YoutRied)




                          1: $3 + 3 - 6 + 1$

                          2: $3 * 3 - (6 + 1)$

                          3: $3 * 3 * 1 - 6$

                          4: $3 * 3 - 6 + 1$

                          5: $(3 * 6) / 3 - 1$

                          6: $(3 * 6) / 3 * 1$

                          7: $(3 * 6) / 3 + 1$

                          8: $3 * 3 - 1 ^ 6$

                          9: $(3 * 6) / (3 - 1)$

                          10: $3 * 3 + 1 ^ 6$

                          11: $36 / 3 - 1$

                          12: $36 / 3 * 1$

                          13: $36 / 3 + 1$

                          14: $3 * 6 - (3 + 1)$

                          15: $3 * 6 - (3 * 1)$

                          16: $3 * 6 - (3 - 1)$

                          17: $3 * 6 - 1 ^ 3$

                          18: $3 * 6 * 1 ^ 3$

                          19: $3 * 6 + 1 ^ 3$

                          20: $3 * 6 + 3 - 1$




                          21-29




                          21: $3 * 6 + 3 * 1$

                          22: $( 1 + 3 ) ! - ( 6 / 3 )$

                          23: $( 1 + 3 ) ! - ( 6 - 3 )$

                          24: $( 6 - 3 / 3 - 1 ) !$

                          25: $1 * 3 ^ 3 - floor(sqrt{6})$

                          26: $( 6 - 3 ) ^ 3 - 1$

                          27: $( 6 - 3 ) ^ 3 * 1$

                          28: $( 6 - 3 ) ^ 3 + 1$

                          29: $31 - 6 / 3$




                          41-50: (Credits to @Bass for 42, 45, 49)




                          41: $ (-1+3!)+36 $

                          42: $ (1+3+3)times 6$

                          43: $ 31 + 6 * ceil(sqrt{3})$

                          44: $floor( 1 * 3 * sqrt{6 ^ 3}) $

                          45: $ 13times3+6$

                          46: $ ceil(sqrt{6 ^ 3} + 31)$

                          47: $ floor(sqrt{sqrt{sqrt{sqrt{sqrt{31!}}}}})+36$

                          48: $6 * ( 3 * 3 - 1 )$

                          49: $13+36$

                          50: $ (6+1)^2 + 3 - 3$




                          51-60:




                          51: $( 3 * 6 - 1 ) * 3$

                          52: $( 3 + 3 + 1 ) * ceil(sqrt{6})$

                          53: $-1+( 3 * 3 * 6 )$

                          54: $ 1*3 * 3 * 6 $

                          55: $1+3*3*6$

                          56: $61-3!+floor(sqrt{3})$

                          57: $1*63-3!$

                          58: $1+63-3!$

                          59: $floor(sqrt{sqrt{sqrt{sqrt{sqrt{6^{(3-1)}}}}}}*3)$

                          60: $(1+3*3)*6$




                          61-70:




                          61: $63-3+1$

                          62: $63+1-ceil(sqrt{3})$

                          63: $63-floor(sqrt{3})+1$

                          64: $63+ceil(sqrt{3})-1$

                          65: $63+3-1$

                          66: $63+3*1$

                          67: $63+3+1$

                          68: $61+3!+floor(sqrt{3})$

                          69: $61+3!+ceil(sqrt{3})$

                          70: $61+3*3$




                          71-80




                          71: $(3+1)!*3-floor(sqrt{sqrt{6}})$

                          72: $(3+1)*3*6$

                          73: $(3+1)!*3+floor(sqrt{sqrt{6}})$

                          74: $(3+1)!*3+floor(sqrt{6})$

                          75: $(3+1)!*3+ceil(sqrt{6})$

                          76: $ceil(sqrt{sqrt{sqrt{ceil(sqrt{sqrt{sqrt{sqrt{sqrt{ceil(sqrt{3!!})!}}}}})}}})*(3*6+1)$

                          76:(much simpler) $13*6-ceil(sqrt{3})$

                          77: $13*6-floor(sqrt{3})$

                          78: $13*floor(sqrt{3})*6$

                          79: $13*6+floor(sqrt{3})$

                          80: $13*6+ceil(sqrt{3})$




                          81-90:




                          81: $(6+3)^{(3-1)}$

                          82: $(6+3)^{ceil(sqrt{3})}+1$








                          share|improve this answer














                          share|improve this answer



                          share|improve this answer








                          edited Jan 28 at 8:36

























                          answered Jan 27 at 23:31









                          Omega KryptonOmega Krypton

                          3,6941338




                          3,6941338












                          • $begingroup$
                            Who said you could use factorials?
                            $endgroup$
                            – Yout Ried
                            Jan 28 at 0:20










                          • $begingroup$
                            What are number 23 (plus you probably can't use factorials and 24? I don't get them.
                            $endgroup$
                            – Yout Ried
                            Jan 28 at 1:08












                          • $begingroup$
                            Oops forgot a ")" and maybe you're missing a factorial for number 24
                            $endgroup$
                            – Yout Ried
                            Jan 28 at 1:16


















                          • $begingroup$
                            Who said you could use factorials?
                            $endgroup$
                            – Yout Ried
                            Jan 28 at 0:20










                          • $begingroup$
                            What are number 23 (plus you probably can't use factorials and 24? I don't get them.
                            $endgroup$
                            – Yout Ried
                            Jan 28 at 1:08












                          • $begingroup$
                            Oops forgot a ")" and maybe you're missing a factorial for number 24
                            $endgroup$
                            – Yout Ried
                            Jan 28 at 1:16
















                          $begingroup$
                          Who said you could use factorials?
                          $endgroup$
                          – Yout Ried
                          Jan 28 at 0:20




                          $begingroup$
                          Who said you could use factorials?
                          $endgroup$
                          – Yout Ried
                          Jan 28 at 0:20












                          $begingroup$
                          What are number 23 (plus you probably can't use factorials and 24? I don't get them.
                          $endgroup$
                          – Yout Ried
                          Jan 28 at 1:08






                          $begingroup$
                          What are number 23 (plus you probably can't use factorials and 24? I don't get them.
                          $endgroup$
                          – Yout Ried
                          Jan 28 at 1:08














                          $begingroup$
                          Oops forgot a ")" and maybe you're missing a factorial for number 24
                          $endgroup$
                          – Yout Ried
                          Jan 28 at 1:16




                          $begingroup$
                          Oops forgot a ")" and maybe you're missing a factorial for number 24
                          $endgroup$
                          – Yout Ried
                          Jan 28 at 1:16











                          0












                          $begingroup$

                          Partial answer 1-50 (w/e 41,47):




                          $1= 1+3+3-6$
                          $2= 1 + (frac{6}{(3+3)})$
                          $3= 1^3+(frac{6}{3})$
                          $4= (frac{6}{3})+3-1$
                          $5= (frac{6}{3})+3^1$
                          $6= 6^1+3-3$
                          $7= 6+1-3+3$
                          $8= 6 + 3 - 1^3$
                          $9= 1^3 * (3+6)$
                          $10= 1^3 +3+6$
                          $11= 13 - (frac{6}{3})$
                          $12= 6+3+3^1$
                          $13= 6+3+3+1$
                          $14= 6*3 - 3 - 1$
                          $15= 6*3 - 3^1$
                          $16= 16 + 3 - 3$
                          $17= 16 + (frac{3}{3})$
                          $18= (frac{6*3}{1^3})$
                          $19= 6*3+1^3$
                          $20= 6*3+3-1$
                          $21= 6*3+3^1$
                          $22= 6*3+3+1$
                          $23= 36-13$
                          $24= 6*(3+1^3)$
                          $25= 16+(3*3)$
                          $26= 13*(frac{6}{3})$
                          $27= 33-6^1$
                          $28= 33-6+1$
                          $29= 31-(frac{6}{3})$
                          $30= 6*(3+3-1)$
                          $31= 13+3*6$
                          $32= 3^3+6-1$
                          $33= (frac{33}{1^6})$
                          $34= 33+1^6$
                          $35= (3+3)*6-1$
                          $36= (3+3)^1*6$
                          $37= 1+(3+3)*6$
                          $38= 33+6-1$
                          $39= 33+6^1$
                          $40= 1+33+6$
                          $41= $
                          $42= (1+3+3)*6$
                          $43= 16+3^3$
                          $44= round(sqrt{6^3}*3^1)$
                          $45= 3*3*(6-1)$
                          $46= ceil(sqrt{6^3}*3)+1$
                          $47= $
                          $48= ((3*3)-1)*6$
                          $49= 16+33$
                          $50= 63-13$







                          share|improve this answer











                          $endgroup$


















                            0












                            $begingroup$

                            Partial answer 1-50 (w/e 41,47):




                            $1= 1+3+3-6$
                            $2= 1 + (frac{6}{(3+3)})$
                            $3= 1^3+(frac{6}{3})$
                            $4= (frac{6}{3})+3-1$
                            $5= (frac{6}{3})+3^1$
                            $6= 6^1+3-3$
                            $7= 6+1-3+3$
                            $8= 6 + 3 - 1^3$
                            $9= 1^3 * (3+6)$
                            $10= 1^3 +3+6$
                            $11= 13 - (frac{6}{3})$
                            $12= 6+3+3^1$
                            $13= 6+3+3+1$
                            $14= 6*3 - 3 - 1$
                            $15= 6*3 - 3^1$
                            $16= 16 + 3 - 3$
                            $17= 16 + (frac{3}{3})$
                            $18= (frac{6*3}{1^3})$
                            $19= 6*3+1^3$
                            $20= 6*3+3-1$
                            $21= 6*3+3^1$
                            $22= 6*3+3+1$
                            $23= 36-13$
                            $24= 6*(3+1^3)$
                            $25= 16+(3*3)$
                            $26= 13*(frac{6}{3})$
                            $27= 33-6^1$
                            $28= 33-6+1$
                            $29= 31-(frac{6}{3})$
                            $30= 6*(3+3-1)$
                            $31= 13+3*6$
                            $32= 3^3+6-1$
                            $33= (frac{33}{1^6})$
                            $34= 33+1^6$
                            $35= (3+3)*6-1$
                            $36= (3+3)^1*6$
                            $37= 1+(3+3)*6$
                            $38= 33+6-1$
                            $39= 33+6^1$
                            $40= 1+33+6$
                            $41= $
                            $42= (1+3+3)*6$
                            $43= 16+3^3$
                            $44= round(sqrt{6^3}*3^1)$
                            $45= 3*3*(6-1)$
                            $46= ceil(sqrt{6^3}*3)+1$
                            $47= $
                            $48= ((3*3)-1)*6$
                            $49= 16+33$
                            $50= 63-13$







                            share|improve this answer











                            $endgroup$
















                              0












                              0








                              0





                              $begingroup$

                              Partial answer 1-50 (w/e 41,47):




                              $1= 1+3+3-6$
                              $2= 1 + (frac{6}{(3+3)})$
                              $3= 1^3+(frac{6}{3})$
                              $4= (frac{6}{3})+3-1$
                              $5= (frac{6}{3})+3^1$
                              $6= 6^1+3-3$
                              $7= 6+1-3+3$
                              $8= 6 + 3 - 1^3$
                              $9= 1^3 * (3+6)$
                              $10= 1^3 +3+6$
                              $11= 13 - (frac{6}{3})$
                              $12= 6+3+3^1$
                              $13= 6+3+3+1$
                              $14= 6*3 - 3 - 1$
                              $15= 6*3 - 3^1$
                              $16= 16 + 3 - 3$
                              $17= 16 + (frac{3}{3})$
                              $18= (frac{6*3}{1^3})$
                              $19= 6*3+1^3$
                              $20= 6*3+3-1$
                              $21= 6*3+3^1$
                              $22= 6*3+3+1$
                              $23= 36-13$
                              $24= 6*(3+1^3)$
                              $25= 16+(3*3)$
                              $26= 13*(frac{6}{3})$
                              $27= 33-6^1$
                              $28= 33-6+1$
                              $29= 31-(frac{6}{3})$
                              $30= 6*(3+3-1)$
                              $31= 13+3*6$
                              $32= 3^3+6-1$
                              $33= (frac{33}{1^6})$
                              $34= 33+1^6$
                              $35= (3+3)*6-1$
                              $36= (3+3)^1*6$
                              $37= 1+(3+3)*6$
                              $38= 33+6-1$
                              $39= 33+6^1$
                              $40= 1+33+6$
                              $41= $
                              $42= (1+3+3)*6$
                              $43= 16+3^3$
                              $44= round(sqrt{6^3}*3^1)$
                              $45= 3*3*(6-1)$
                              $46= ceil(sqrt{6^3}*3)+1$
                              $47= $
                              $48= ((3*3)-1)*6$
                              $49= 16+33$
                              $50= 63-13$







                              share|improve this answer











                              $endgroup$



                              Partial answer 1-50 (w/e 41,47):




                              $1= 1+3+3-6$
                              $2= 1 + (frac{6}{(3+3)})$
                              $3= 1^3+(frac{6}{3})$
                              $4= (frac{6}{3})+3-1$
                              $5= (frac{6}{3})+3^1$
                              $6= 6^1+3-3$
                              $7= 6+1-3+3$
                              $8= 6 + 3 - 1^3$
                              $9= 1^3 * (3+6)$
                              $10= 1^3 +3+6$
                              $11= 13 - (frac{6}{3})$
                              $12= 6+3+3^1$
                              $13= 6+3+3+1$
                              $14= 6*3 - 3 - 1$
                              $15= 6*3 - 3^1$
                              $16= 16 + 3 - 3$
                              $17= 16 + (frac{3}{3})$
                              $18= (frac{6*3}{1^3})$
                              $19= 6*3+1^3$
                              $20= 6*3+3-1$
                              $21= 6*3+3^1$
                              $22= 6*3+3+1$
                              $23= 36-13$
                              $24= 6*(3+1^3)$
                              $25= 16+(3*3)$
                              $26= 13*(frac{6}{3})$
                              $27= 33-6^1$
                              $28= 33-6+1$
                              $29= 31-(frac{6}{3})$
                              $30= 6*(3+3-1)$
                              $31= 13+3*6$
                              $32= 3^3+6-1$
                              $33= (frac{33}{1^6})$
                              $34= 33+1^6$
                              $35= (3+3)*6-1$
                              $36= (3+3)^1*6$
                              $37= 1+(3+3)*6$
                              $38= 33+6-1$
                              $39= 33+6^1$
                              $40= 1+33+6$
                              $41= $
                              $42= (1+3+3)*6$
                              $43= 16+3^3$
                              $44= round(sqrt{6^3}*3^1)$
                              $45= 3*3*(6-1)$
                              $46= ceil(sqrt{6^3}*3)+1$
                              $47= $
                              $48= ((3*3)-1)*6$
                              $49= 16+33$
                              $50= 63-13$








                              share|improve this answer














                              share|improve this answer



                              share|improve this answer








                              edited Jan 28 at 3:39

























                              answered Jan 28 at 3:01









                              MukyuuMukyuu

                              340112




                              340112























                                  0












                                  $begingroup$

                                  Alrighty, I'm piggybacking off of @OmegaKrypton else and adding some of my own.



                                  1 to 10




                                  1: $1 + 3 + 3 - 6$

                                  2: $(1 + 3) times 3 / 6$

                                  3: $1^3 +3/6$

                                  4: $13 - 3 - 6$

                                  5: $-1^{33} +6$

                                  6: $1times3-3+6$

                                  7: $ 1 + 3 -3 +6$

                                  8: $ 1+3/3 + 6$

                                  9: $ 1^3 times (3+6)$

                                  10: $ 1^3 + 3+6$




                                  11 to 20




                                  11: $ sqrt{1+3}+3+6$

                                  12: $1times 3 + 3 + 6$

                                  13: $1 + 3+3+6$

                                  14: $-1 + 3times 3+6$

                                  15: $-1times3 + 3times 6$

                                  16: $1 - 3 + 3 times 6$

                                  17: $ -1^3 +3times 6$

                                  18: $ (1+3)*3+6 $

                                  19: $13 + sqrt{36}$

                                  20: $-1 + 3^3 - 6$




                                  21 to 30




                                  21: $ 1 * 3^3 - 6 $

                                  22: $ 13 + 3 + 6$

                                  23: $ -13+36 $

                                  24: $ (1+3)timessqrt{36}$

                                  25: $ 3*6+3!-1$ or $ (6-1)^(3!/3)$

                                  26: $ -1+33-6$

                                  27: $ 1*33-6 $

                                  28: $ 1+33-6$

                                  29: $ 36-3!-1$

                                  30: $ (-1+3+3)times 6$




                                  31 to 40




                                  31: $ 13+3*6 $

                                  32: $ -1+3^3+6$

                                  33: $ 13*3-6 $

                                  34: $ 1+3^3+6$

                                  35: $ -1+(3+3)times6 $

                                  36: $ 1times(3+3)times 6$

                                  37: $ 1^3+36$

                                  38: $ sqrt{1+3}+36$

                                  39: $ 1times3 + 36$

                                  40: $ 1+33+6$




                                  41 to 50




                                  41: $ $

                                  42: $ (1+3+3)times 6$

                                  43: $ 3^3+16$

                                  44: $ $

                                  45: $ 13times3+6$

                                  46: $ $

                                  47: $ $

                                  48: $ 16*(3!-3)$

                                  49: $13+36$

                                  50: $ 63-13$




                                  I added a few. It's getting late here; will come back tomorrow.






                                  share|improve this answer











                                  $endgroup$


















                                    0












                                    $begingroup$

                                    Alrighty, I'm piggybacking off of @OmegaKrypton else and adding some of my own.



                                    1 to 10




                                    1: $1 + 3 + 3 - 6$

                                    2: $(1 + 3) times 3 / 6$

                                    3: $1^3 +3/6$

                                    4: $13 - 3 - 6$

                                    5: $-1^{33} +6$

                                    6: $1times3-3+6$

                                    7: $ 1 + 3 -3 +6$

                                    8: $ 1+3/3 + 6$

                                    9: $ 1^3 times (3+6)$

                                    10: $ 1^3 + 3+6$




                                    11 to 20




                                    11: $ sqrt{1+3}+3+6$

                                    12: $1times 3 + 3 + 6$

                                    13: $1 + 3+3+6$

                                    14: $-1 + 3times 3+6$

                                    15: $-1times3 + 3times 6$

                                    16: $1 - 3 + 3 times 6$

                                    17: $ -1^3 +3times 6$

                                    18: $ (1+3)*3+6 $

                                    19: $13 + sqrt{36}$

                                    20: $-1 + 3^3 - 6$




                                    21 to 30




                                    21: $ 1 * 3^3 - 6 $

                                    22: $ 13 + 3 + 6$

                                    23: $ -13+36 $

                                    24: $ (1+3)timessqrt{36}$

                                    25: $ 3*6+3!-1$ or $ (6-1)^(3!/3)$

                                    26: $ -1+33-6$

                                    27: $ 1*33-6 $

                                    28: $ 1+33-6$

                                    29: $ 36-3!-1$

                                    30: $ (-1+3+3)times 6$




                                    31 to 40




                                    31: $ 13+3*6 $

                                    32: $ -1+3^3+6$

                                    33: $ 13*3-6 $

                                    34: $ 1+3^3+6$

                                    35: $ -1+(3+3)times6 $

                                    36: $ 1times(3+3)times 6$

                                    37: $ 1^3+36$

                                    38: $ sqrt{1+3}+36$

                                    39: $ 1times3 + 36$

                                    40: $ 1+33+6$




                                    41 to 50




                                    41: $ $

                                    42: $ (1+3+3)times 6$

                                    43: $ 3^3+16$

                                    44: $ $

                                    45: $ 13times3+6$

                                    46: $ $

                                    47: $ $

                                    48: $ 16*(3!-3)$

                                    49: $13+36$

                                    50: $ 63-13$




                                    I added a few. It's getting late here; will come back tomorrow.






                                    share|improve this answer











                                    $endgroup$
















                                      0












                                      0








                                      0





                                      $begingroup$

                                      Alrighty, I'm piggybacking off of @OmegaKrypton else and adding some of my own.



                                      1 to 10




                                      1: $1 + 3 + 3 - 6$

                                      2: $(1 + 3) times 3 / 6$

                                      3: $1^3 +3/6$

                                      4: $13 - 3 - 6$

                                      5: $-1^{33} +6$

                                      6: $1times3-3+6$

                                      7: $ 1 + 3 -3 +6$

                                      8: $ 1+3/3 + 6$

                                      9: $ 1^3 times (3+6)$

                                      10: $ 1^3 + 3+6$




                                      11 to 20




                                      11: $ sqrt{1+3}+3+6$

                                      12: $1times 3 + 3 + 6$

                                      13: $1 + 3+3+6$

                                      14: $-1 + 3times 3+6$

                                      15: $-1times3 + 3times 6$

                                      16: $1 - 3 + 3 times 6$

                                      17: $ -1^3 +3times 6$

                                      18: $ (1+3)*3+6 $

                                      19: $13 + sqrt{36}$

                                      20: $-1 + 3^3 - 6$




                                      21 to 30




                                      21: $ 1 * 3^3 - 6 $

                                      22: $ 13 + 3 + 6$

                                      23: $ -13+36 $

                                      24: $ (1+3)timessqrt{36}$

                                      25: $ 3*6+3!-1$ or $ (6-1)^(3!/3)$

                                      26: $ -1+33-6$

                                      27: $ 1*33-6 $

                                      28: $ 1+33-6$

                                      29: $ 36-3!-1$

                                      30: $ (-1+3+3)times 6$




                                      31 to 40




                                      31: $ 13+3*6 $

                                      32: $ -1+3^3+6$

                                      33: $ 13*3-6 $

                                      34: $ 1+3^3+6$

                                      35: $ -1+(3+3)times6 $

                                      36: $ 1times(3+3)times 6$

                                      37: $ 1^3+36$

                                      38: $ sqrt{1+3}+36$

                                      39: $ 1times3 + 36$

                                      40: $ 1+33+6$




                                      41 to 50




                                      41: $ $

                                      42: $ (1+3+3)times 6$

                                      43: $ 3^3+16$

                                      44: $ $

                                      45: $ 13times3+6$

                                      46: $ $

                                      47: $ $

                                      48: $ 16*(3!-3)$

                                      49: $13+36$

                                      50: $ 63-13$




                                      I added a few. It's getting late here; will come back tomorrow.






                                      share|improve this answer











                                      $endgroup$



                                      Alrighty, I'm piggybacking off of @OmegaKrypton else and adding some of my own.



                                      1 to 10




                                      1: $1 + 3 + 3 - 6$

                                      2: $(1 + 3) times 3 / 6$

                                      3: $1^3 +3/6$

                                      4: $13 - 3 - 6$

                                      5: $-1^{33} +6$

                                      6: $1times3-3+6$

                                      7: $ 1 + 3 -3 +6$

                                      8: $ 1+3/3 + 6$

                                      9: $ 1^3 times (3+6)$

                                      10: $ 1^3 + 3+6$




                                      11 to 20




                                      11: $ sqrt{1+3}+3+6$

                                      12: $1times 3 + 3 + 6$

                                      13: $1 + 3+3+6$

                                      14: $-1 + 3times 3+6$

                                      15: $-1times3 + 3times 6$

                                      16: $1 - 3 + 3 times 6$

                                      17: $ -1^3 +3times 6$

                                      18: $ (1+3)*3+6 $

                                      19: $13 + sqrt{36}$

                                      20: $-1 + 3^3 - 6$




                                      21 to 30




                                      21: $ 1 * 3^3 - 6 $

                                      22: $ 13 + 3 + 6$

                                      23: $ -13+36 $

                                      24: $ (1+3)timessqrt{36}$

                                      25: $ 3*6+3!-1$ or $ (6-1)^(3!/3)$

                                      26: $ -1+33-6$

                                      27: $ 1*33-6 $

                                      28: $ 1+33-6$

                                      29: $ 36-3!-1$

                                      30: $ (-1+3+3)times 6$




                                      31 to 40




                                      31: $ 13+3*6 $

                                      32: $ -1+3^3+6$

                                      33: $ 13*3-6 $

                                      34: $ 1+3^3+6$

                                      35: $ -1+(3+3)times6 $

                                      36: $ 1times(3+3)times 6$

                                      37: $ 1^3+36$

                                      38: $ sqrt{1+3}+36$

                                      39: $ 1times3 + 36$

                                      40: $ 1+33+6$




                                      41 to 50




                                      41: $ $

                                      42: $ (1+3+3)times 6$

                                      43: $ 3^3+16$

                                      44: $ $

                                      45: $ 13times3+6$

                                      46: $ $

                                      47: $ $

                                      48: $ 16*(3!-3)$

                                      49: $13+36$

                                      50: $ 63-13$




                                      I added a few. It's getting late here; will come back tomorrow.







                                      share|improve this answer














                                      share|improve this answer



                                      share|improve this answer








                                      edited Jan 28 at 6:12









                                      Omega Krypton

                                      3,6941338




                                      3,6941338










                                      answered Jan 28 at 2:58









                                      Brandon_JBrandon_J

                                      1,28427




                                      1,28427























                                          0












                                          $begingroup$

                                          Expanding on Bass's answer, I added some new numbers.



                                          (I lost track on which numbers I added, though 1-40 is all Bass)




                                          1: $1 + 3 + 3 - 6$

                                          2: $(1 + 3) times 3 / 6$

                                          3: $1*3 * 3 - 6$

                                          4: $13 - 3 - 6$

                                          5: $-1^{33} +6$

                                          6: $1times3-3+6$

                                          7: $ 1 + 3 -3 +6$

                                          8: $ 1+3/3 + 6$

                                          9: $ 1^3 times (3+6)$

                                          10: $ 1 + sqrt[3]3^6$
                                          11: $ sqrt{1+3}+3+6$

                                          12: $1times 3 + 3 + 6$

                                          13: $1 + 3+3+6$

                                          14: $-1 + 3times 3+6$

                                          15: $-1times3 + 3times 6$

                                          16: $1 - 3 + 3 times 6$

                                          17: $ -1^3 +3times 6$

                                          18: $ (1+3)*3+6 $

                                          19: $13 + sqrt{36}$

                                          20: $-1 + 3^3 - 6$

                                          21: $ 1 * 3^3 - 6 $

                                          22: $ 13 + 3 + 6$

                                          23: $ -13+36 $

                                          24: $ (1+3)timessqrt{36}$

                                          25: $ 1 - 3 + sqrt3^6$

                                          26: $ -1+33-6$

                                          27: $ 1*33-6 $

                                          28: $ 1+33-6$

                                          29: $ -1 + 3 + sqrt3^6$

                                          30: $ (-1+3+3)times 6$

                                          31: $ 13+3*6 $

                                          32: $ -1+3^3+6$

                                          33: $ 13*3-6 $

                                          34: $ 1+3^3+6$

                                          35: $ -1+(3+3)times6 $

                                          36: $ 1times(3+3)times 6$

                                          37: $ 1^3+36$

                                          38: $ sqrt{1+3}+36$

                                          39: $ 1times3 + 36$

                                          40: $ 1+33+6$

                                          41: $ $

                                          42: $ (1+3+3)times 6$

                                          43: $ 3^3 + 16 $

                                          44: $ $

                                          45: $ 13times3+6$

                                          46: $ $

                                          47: $ $

                                          48: $ (-1 + 3 times 3) times 6 $

                                          49: $13+36$

                                          50: $ $

                                          51: $ 16*3+3 $

                                          52: $ 61 - 3times3$

                                          53: $ -1 +3 times 3 times 6$

                                          54: $ 1times 3 times 3 times 6$

                                          55: $ 1 + 3 times 3 times 6$

                                          56: $ $

                                          57: $ (6times3+1)times3$

                                          58: $ (1+3)^3-6$

                                          59: $ 63 - 3 - 1$

                                          60: $ (1+3times3)times6$

                                          61: $ 63 - 3 + 1$

                                          62: $ 63 - 1^3$

                                          63: $ 63 * 1^3$

                                          64: $ (1+3/3)^6$

                                          65: $ 63 + 3 - 1$

                                          66: $ 1 times (63 + 3) $

                                          67: $ 63 + 3 + 1$

                                          68: $ $

                                          69: $ $

                                          70: $ (1+3)^3+6 $
                                          71: $ 6^3 / 3 - 1 $

                                          72: $ (1+3)times 3 times 6$

                                          73: $ 6^3 / 3 + 1 $

                                          74: $ $

                                          75: $ 3^(3+1) - 6$

                                          76: $ 63+13 $

                                          77: $ $

                                          78: $ 13 times sqrt{36}$

                                          79: $ $

                                          80: $ -1 + 3timessqrt3^6$

                                          81: $ 1times3timessqrt3^6$

                                          82: $ 1+3timessqrt3^6$

                                          83: $ $

                                          84: $ $

                                          85: $ $

                                          86: $ $

                                          87: $ 3^(3+1) + 6$

                                          88: $ 61 + 3^3 $

                                          89: $ $

                                          90: $ $

                                          91: $ $

                                          92: $ $

                                          93: $ $

                                          94: $ 33 + 61 $

                                          95: $ $

                                          96: $ (13+3)times6 $

                                          97: $ $

                                          98: $ $

                                          99: $ 31times3+6$

                                          100: $ $




                                          Only need 41, 44, 46, 47, 50, 56, 68, 69, 74, 77, 79, 83, 84, 85, 86, 89, 90, 91, 92, 93, 94, 95, 97, 98, and 100 now!






                                          share|improve this answer










                                          New contributor




                                          Embodiment of Ignorance is a new contributor to this site. Take care in asking for clarification, commenting, and answering.
                                          Check out our Code of Conduct.






                                          $endgroup$













                                          • $begingroup$
                                            Wow that is A LOT of numbers
                                            $endgroup$
                                            – North
                                            Jan 30 at 3:07
















                                          0












                                          $begingroup$

                                          Expanding on Bass's answer, I added some new numbers.



                                          (I lost track on which numbers I added, though 1-40 is all Bass)




                                          1: $1 + 3 + 3 - 6$

                                          2: $(1 + 3) times 3 / 6$

                                          3: $1*3 * 3 - 6$

                                          4: $13 - 3 - 6$

                                          5: $-1^{33} +6$

                                          6: $1times3-3+6$

                                          7: $ 1 + 3 -3 +6$

                                          8: $ 1+3/3 + 6$

                                          9: $ 1^3 times (3+6)$

                                          10: $ 1 + sqrt[3]3^6$
                                          11: $ sqrt{1+3}+3+6$

                                          12: $1times 3 + 3 + 6$

                                          13: $1 + 3+3+6$

                                          14: $-1 + 3times 3+6$

                                          15: $-1times3 + 3times 6$

                                          16: $1 - 3 + 3 times 6$

                                          17: $ -1^3 +3times 6$

                                          18: $ (1+3)*3+6 $

                                          19: $13 + sqrt{36}$

                                          20: $-1 + 3^3 - 6$

                                          21: $ 1 * 3^3 - 6 $

                                          22: $ 13 + 3 + 6$

                                          23: $ -13+36 $

                                          24: $ (1+3)timessqrt{36}$

                                          25: $ 1 - 3 + sqrt3^6$

                                          26: $ -1+33-6$

                                          27: $ 1*33-6 $

                                          28: $ 1+33-6$

                                          29: $ -1 + 3 + sqrt3^6$

                                          30: $ (-1+3+3)times 6$

                                          31: $ 13+3*6 $

                                          32: $ -1+3^3+6$

                                          33: $ 13*3-6 $

                                          34: $ 1+3^3+6$

                                          35: $ -1+(3+3)times6 $

                                          36: $ 1times(3+3)times 6$

                                          37: $ 1^3+36$

                                          38: $ sqrt{1+3}+36$

                                          39: $ 1times3 + 36$

                                          40: $ 1+33+6$

                                          41: $ $

                                          42: $ (1+3+3)times 6$

                                          43: $ 3^3 + 16 $

                                          44: $ $

                                          45: $ 13times3+6$

                                          46: $ $

                                          47: $ $

                                          48: $ (-1 + 3 times 3) times 6 $

                                          49: $13+36$

                                          50: $ $

                                          51: $ 16*3+3 $

                                          52: $ 61 - 3times3$

                                          53: $ -1 +3 times 3 times 6$

                                          54: $ 1times 3 times 3 times 6$

                                          55: $ 1 + 3 times 3 times 6$

                                          56: $ $

                                          57: $ (6times3+1)times3$

                                          58: $ (1+3)^3-6$

                                          59: $ 63 - 3 - 1$

                                          60: $ (1+3times3)times6$

                                          61: $ 63 - 3 + 1$

                                          62: $ 63 - 1^3$

                                          63: $ 63 * 1^3$

                                          64: $ (1+3/3)^6$

                                          65: $ 63 + 3 - 1$

                                          66: $ 1 times (63 + 3) $

                                          67: $ 63 + 3 + 1$

                                          68: $ $

                                          69: $ $

                                          70: $ (1+3)^3+6 $
                                          71: $ 6^3 / 3 - 1 $

                                          72: $ (1+3)times 3 times 6$

                                          73: $ 6^3 / 3 + 1 $

                                          74: $ $

                                          75: $ 3^(3+1) - 6$

                                          76: $ 63+13 $

                                          77: $ $

                                          78: $ 13 times sqrt{36}$

                                          79: $ $

                                          80: $ -1 + 3timessqrt3^6$

                                          81: $ 1times3timessqrt3^6$

                                          82: $ 1+3timessqrt3^6$

                                          83: $ $

                                          84: $ $

                                          85: $ $

                                          86: $ $

                                          87: $ 3^(3+1) + 6$

                                          88: $ 61 + 3^3 $

                                          89: $ $

                                          90: $ $

                                          91: $ $

                                          92: $ $

                                          93: $ $

                                          94: $ 33 + 61 $

                                          95: $ $

                                          96: $ (13+3)times6 $

                                          97: $ $

                                          98: $ $

                                          99: $ 31times3+6$

                                          100: $ $




                                          Only need 41, 44, 46, 47, 50, 56, 68, 69, 74, 77, 79, 83, 84, 85, 86, 89, 90, 91, 92, 93, 94, 95, 97, 98, and 100 now!






                                          share|improve this answer










                                          New contributor




                                          Embodiment of Ignorance is a new contributor to this site. Take care in asking for clarification, commenting, and answering.
                                          Check out our Code of Conduct.






                                          $endgroup$













                                          • $begingroup$
                                            Wow that is A LOT of numbers
                                            $endgroup$
                                            – North
                                            Jan 30 at 3:07














                                          0












                                          0








                                          0





                                          $begingroup$

                                          Expanding on Bass's answer, I added some new numbers.



                                          (I lost track on which numbers I added, though 1-40 is all Bass)




                                          1: $1 + 3 + 3 - 6$

                                          2: $(1 + 3) times 3 / 6$

                                          3: $1*3 * 3 - 6$

                                          4: $13 - 3 - 6$

                                          5: $-1^{33} +6$

                                          6: $1times3-3+6$

                                          7: $ 1 + 3 -3 +6$

                                          8: $ 1+3/3 + 6$

                                          9: $ 1^3 times (3+6)$

                                          10: $ 1 + sqrt[3]3^6$
                                          11: $ sqrt{1+3}+3+6$

                                          12: $1times 3 + 3 + 6$

                                          13: $1 + 3+3+6$

                                          14: $-1 + 3times 3+6$

                                          15: $-1times3 + 3times 6$

                                          16: $1 - 3 + 3 times 6$

                                          17: $ -1^3 +3times 6$

                                          18: $ (1+3)*3+6 $

                                          19: $13 + sqrt{36}$

                                          20: $-1 + 3^3 - 6$

                                          21: $ 1 * 3^3 - 6 $

                                          22: $ 13 + 3 + 6$

                                          23: $ -13+36 $

                                          24: $ (1+3)timessqrt{36}$

                                          25: $ 1 - 3 + sqrt3^6$

                                          26: $ -1+33-6$

                                          27: $ 1*33-6 $

                                          28: $ 1+33-6$

                                          29: $ -1 + 3 + sqrt3^6$

                                          30: $ (-1+3+3)times 6$

                                          31: $ 13+3*6 $

                                          32: $ -1+3^3+6$

                                          33: $ 13*3-6 $

                                          34: $ 1+3^3+6$

                                          35: $ -1+(3+3)times6 $

                                          36: $ 1times(3+3)times 6$

                                          37: $ 1^3+36$

                                          38: $ sqrt{1+3}+36$

                                          39: $ 1times3 + 36$

                                          40: $ 1+33+6$

                                          41: $ $

                                          42: $ (1+3+3)times 6$

                                          43: $ 3^3 + 16 $

                                          44: $ $

                                          45: $ 13times3+6$

                                          46: $ $

                                          47: $ $

                                          48: $ (-1 + 3 times 3) times 6 $

                                          49: $13+36$

                                          50: $ $

                                          51: $ 16*3+3 $

                                          52: $ 61 - 3times3$

                                          53: $ -1 +3 times 3 times 6$

                                          54: $ 1times 3 times 3 times 6$

                                          55: $ 1 + 3 times 3 times 6$

                                          56: $ $

                                          57: $ (6times3+1)times3$

                                          58: $ (1+3)^3-6$

                                          59: $ 63 - 3 - 1$

                                          60: $ (1+3times3)times6$

                                          61: $ 63 - 3 + 1$

                                          62: $ 63 - 1^3$

                                          63: $ 63 * 1^3$

                                          64: $ (1+3/3)^6$

                                          65: $ 63 + 3 - 1$

                                          66: $ 1 times (63 + 3) $

                                          67: $ 63 + 3 + 1$

                                          68: $ $

                                          69: $ $

                                          70: $ (1+3)^3+6 $
                                          71: $ 6^3 / 3 - 1 $

                                          72: $ (1+3)times 3 times 6$

                                          73: $ 6^3 / 3 + 1 $

                                          74: $ $

                                          75: $ 3^(3+1) - 6$

                                          76: $ 63+13 $

                                          77: $ $

                                          78: $ 13 times sqrt{36}$

                                          79: $ $

                                          80: $ -1 + 3timessqrt3^6$

                                          81: $ 1times3timessqrt3^6$

                                          82: $ 1+3timessqrt3^6$

                                          83: $ $

                                          84: $ $

                                          85: $ $

                                          86: $ $

                                          87: $ 3^(3+1) + 6$

                                          88: $ 61 + 3^3 $

                                          89: $ $

                                          90: $ $

                                          91: $ $

                                          92: $ $

                                          93: $ $

                                          94: $ 33 + 61 $

                                          95: $ $

                                          96: $ (13+3)times6 $

                                          97: $ $

                                          98: $ $

                                          99: $ 31times3+6$

                                          100: $ $




                                          Only need 41, 44, 46, 47, 50, 56, 68, 69, 74, 77, 79, 83, 84, 85, 86, 89, 90, 91, 92, 93, 94, 95, 97, 98, and 100 now!






                                          share|improve this answer










                                          New contributor




                                          Embodiment of Ignorance is a new contributor to this site. Take care in asking for clarification, commenting, and answering.
                                          Check out our Code of Conduct.






                                          $endgroup$



                                          Expanding on Bass's answer, I added some new numbers.



                                          (I lost track on which numbers I added, though 1-40 is all Bass)




                                          1: $1 + 3 + 3 - 6$

                                          2: $(1 + 3) times 3 / 6$

                                          3: $1*3 * 3 - 6$

                                          4: $13 - 3 - 6$

                                          5: $-1^{33} +6$

                                          6: $1times3-3+6$

                                          7: $ 1 + 3 -3 +6$

                                          8: $ 1+3/3 + 6$

                                          9: $ 1^3 times (3+6)$

                                          10: $ 1 + sqrt[3]3^6$
                                          11: $ sqrt{1+3}+3+6$

                                          12: $1times 3 + 3 + 6$

                                          13: $1 + 3+3+6$

                                          14: $-1 + 3times 3+6$

                                          15: $-1times3 + 3times 6$

                                          16: $1 - 3 + 3 times 6$

                                          17: $ -1^3 +3times 6$

                                          18: $ (1+3)*3+6 $

                                          19: $13 + sqrt{36}$

                                          20: $-1 + 3^3 - 6$

                                          21: $ 1 * 3^3 - 6 $

                                          22: $ 13 + 3 + 6$

                                          23: $ -13+36 $

                                          24: $ (1+3)timessqrt{36}$

                                          25: $ 1 - 3 + sqrt3^6$

                                          26: $ -1+33-6$

                                          27: $ 1*33-6 $

                                          28: $ 1+33-6$

                                          29: $ -1 + 3 + sqrt3^6$

                                          30: $ (-1+3+3)times 6$

                                          31: $ 13+3*6 $

                                          32: $ -1+3^3+6$

                                          33: $ 13*3-6 $

                                          34: $ 1+3^3+6$

                                          35: $ -1+(3+3)times6 $

                                          36: $ 1times(3+3)times 6$

                                          37: $ 1^3+36$

                                          38: $ sqrt{1+3}+36$

                                          39: $ 1times3 + 36$

                                          40: $ 1+33+6$

                                          41: $ $

                                          42: $ (1+3+3)times 6$

                                          43: $ 3^3 + 16 $

                                          44: $ $

                                          45: $ 13times3+6$

                                          46: $ $

                                          47: $ $

                                          48: $ (-1 + 3 times 3) times 6 $

                                          49: $13+36$

                                          50: $ $

                                          51: $ 16*3+3 $

                                          52: $ 61 - 3times3$

                                          53: $ -1 +3 times 3 times 6$

                                          54: $ 1times 3 times 3 times 6$

                                          55: $ 1 + 3 times 3 times 6$

                                          56: $ $

                                          57: $ (6times3+1)times3$

                                          58: $ (1+3)^3-6$

                                          59: $ 63 - 3 - 1$

                                          60: $ (1+3times3)times6$

                                          61: $ 63 - 3 + 1$

                                          62: $ 63 - 1^3$

                                          63: $ 63 * 1^3$

                                          64: $ (1+3/3)^6$

                                          65: $ 63 + 3 - 1$

                                          66: $ 1 times (63 + 3) $

                                          67: $ 63 + 3 + 1$

                                          68: $ $

                                          69: $ $

                                          70: $ (1+3)^3+6 $
                                          71: $ 6^3 / 3 - 1 $

                                          72: $ (1+3)times 3 times 6$

                                          73: $ 6^3 / 3 + 1 $

                                          74: $ $

                                          75: $ 3^(3+1) - 6$

                                          76: $ 63+13 $

                                          77: $ $

                                          78: $ 13 times sqrt{36}$

                                          79: $ $

                                          80: $ -1 + 3timessqrt3^6$

                                          81: $ 1times3timessqrt3^6$

                                          82: $ 1+3timessqrt3^6$

                                          83: $ $

                                          84: $ $

                                          85: $ $

                                          86: $ $

                                          87: $ 3^(3+1) + 6$

                                          88: $ 61 + 3^3 $

                                          89: $ $

                                          90: $ $

                                          91: $ $

                                          92: $ $

                                          93: $ $

                                          94: $ 33 + 61 $

                                          95: $ $

                                          96: $ (13+3)times6 $

                                          97: $ $

                                          98: $ $

                                          99: $ 31times3+6$

                                          100: $ $




                                          Only need 41, 44, 46, 47, 50, 56, 68, 69, 74, 77, 79, 83, 84, 85, 86, 89, 90, 91, 92, 93, 94, 95, 97, 98, and 100 now!







                                          share|improve this answer










                                          New contributor




                                          Embodiment of Ignorance is a new contributor to this site. Take care in asking for clarification, commenting, and answering.
                                          Check out our Code of Conduct.









                                          share|improve this answer



                                          share|improve this answer








                                          edited Jan 30 at 3:49





















                                          New contributor




                                          Embodiment of Ignorance is a new contributor to this site. Take care in asking for clarification, commenting, and answering.
                                          Check out our Code of Conduct.









                                          answered Jan 30 at 2:56









                                          Embodiment of IgnoranceEmbodiment of Ignorance

                                          1096




                                          1096




                                          New contributor




                                          Embodiment of Ignorance is a new contributor to this site. Take care in asking for clarification, commenting, and answering.
                                          Check out our Code of Conduct.





                                          New contributor





                                          Embodiment of Ignorance is a new contributor to this site. Take care in asking for clarification, commenting, and answering.
                                          Check out our Code of Conduct.






                                          Embodiment of Ignorance is a new contributor to this site. Take care in asking for clarification, commenting, and answering.
                                          Check out our Code of Conduct.












                                          • $begingroup$
                                            Wow that is A LOT of numbers
                                            $endgroup$
                                            – North
                                            Jan 30 at 3:07


















                                          • $begingroup$
                                            Wow that is A LOT of numbers
                                            $endgroup$
                                            – North
                                            Jan 30 at 3:07
















                                          $begingroup$
                                          Wow that is A LOT of numbers
                                          $endgroup$
                                          – North
                                          Jan 30 at 3:07




                                          $begingroup$
                                          Wow that is A LOT of numbers
                                          $endgroup$
                                          – North
                                          Jan 30 at 3:07


















                                          draft saved

                                          draft discarded




















































                                          Thanks for contributing an answer to Puzzling Stack Exchange!


                                          • Please be sure to answer the question. Provide details and share your research!

                                          But avoid



                                          • Asking for help, clarification, or responding to other answers.

                                          • Making statements based on opinion; back them up with references or personal experience.


                                          Use MathJax to format equations. MathJax reference.


                                          To learn more, see our tips on writing great answers.




                                          draft saved


                                          draft discarded














                                          StackExchange.ready(
                                          function () {
                                          StackExchange.openid.initPostLogin('.new-post-login', 'https%3a%2f%2fpuzzling.stackexchange.com%2fquestions%2f78918%2fcreate-all-numbers-from-1-100-using-1-3-3-6%23new-answer', 'question_page');
                                          }
                                          );

                                          Post as a guest















                                          Required, but never shown





















































                                          Required, but never shown














                                          Required, but never shown












                                          Required, but never shown







                                          Required, but never shown

































                                          Required, but never shown














                                          Required, but never shown












                                          Required, but never shown







                                          Required, but never shown







                                          Popular posts from this blog

                                          Biblatex bibliography style without URLs when DOI exists (in Overleaf with Zotero bibliography)

                                          ComboBox Display Member on multiple fields

                                          Is it possible to collect Nectar points via Trainline?